7225 Exam 2

¡Supera tus tareas y exámenes ahora con Quizwiz!

36-42 months language

Responds to simple three-part commands, understands "just one", gives full name when asked, lots of questions, begins to relate events

Treatment for GABHS: what to use and why

"Regular" strength amoxicillin 40-50mg/kg/day. Can give in one dose daily X 7 to 10 days. Or Penicillin VK Usually Group A strep is usually a self-limited illness lasting 2-3 days Primary rationale for treatment: Prevent secondary complication e.g. retropharyngeal abscess. Prevents rheumatic fever if given within 7 to 9 days of infection. Comfort

enzyme-linked immunosorbent assay (ELISA)

"wet lab" type biochemical assay. antigens from the same are attached to a surface. a further specific antibody is applied over the surface so it can bind to the antigen. the antibody is linked to an enzyme, a substance containing the enzyme substrate is added causing color change.

The five diseases plus one 1st, 2nd, 3rd, 4th, 5th, 6th

1st: rubeola or measles 2nd: scarlet fever 3rd: rubella, german measles, 3-day measles 4th: fialow-dukes disease, staph scalded skin 5th: erythema infectiosum (parvovirus) 6th: roseola infants or exanthema subitem

48-60 months language

Responds to three-stop commands, asks "how" questions, can use conjunctions to combine words and phrases, answers verbally to questions, such as "How are you?"

Successful breastfeeding stool output

24-hour to 2 days: 2 meconium stools Day 3: 2 to 3 stools. Loose, yellow, seedy First 2 weeks: number may increase; some stool with every feeding After 1 month: pattern may change; less frequent; may go several days between stools No problem if infant is healthy and gaining weight. Problem if infrequent stooling in the first month

Pityriasis (tinea) versicolor

Symptoms: none color: hyperpigmented Distribution: upper chest, neck, and shoulders blanching: no Pruritic: no Texture: plaques, slightly raised with scaly borders

Bacteremia risk

30-50% of those who are ultimately diagnosed with bacterial meningitis have been seen by a physician within the prior week (usually 1-2 days before). At the clinic, they were diagnosed as having a trivial illness and frequently treated with oral antibiotics.

Etiologies for newborn jaundice:

60-80% of newborns have jaundice. Unconjugated: bilirubin overproduction, hemolysis, isoimmune hemolytic disease, hemoglobinopathies or RBC membrane defects, G6PD deficiency, cephalohematoma or significant bruising, polycythemia. Impaired bilirubin uptake: increased enterohepatic circulation, breastfeeding jaundice, intestinal obstruction/ileus, poor blood flow, sepsis, congestive heart failure. Impaired bilirubin conjugation: crigler-naijar types 1 and 2, gilbert syndrome, breast milk jaundice.

Herpetic gingivostomatitis

6mos to 5 years most common presentation of HSV. looks like grouped vesicles with erythematous base, very painful, high fever, drooling, cervical lymphadenopathy. Supportive treatment. (Very narrow window to start antiviral therapy of acyclovir, within 48 hours)

Treatment for bacteremia relative indications

<6 months or not completely immunized child <36 months. CBC, UA, blood and urine cultures. CXR when WBC >20,000 of presence of respiratory symtoms Consider IV ceftriaxone and amoxicillin or macrolides or clindamycin. Girls <24mos consider UA/culture by algorithm, uncircumcised boys less than 12 months and all male infants 6 months consider a UA and culture by algorithm.

Metatarsus adductus (MTA): What is it? treatment?

A positional foot deformity related to intrauterine packaging. Forefoot compressed into adduction during intrauterine development. May be passively correctable or rigid 1 to 100 infants. Ankle motion is normal Assess flexibility of forefoot by holding heel in neutral position and abducting forefoot. Flexible deformity will straighten and rigid deformity will not. Check hips. Treatment: 90% are flexible and correct on their own over the first few years of life → refer to PT if needed. May do passive stretching (abduct forefoot) with diaper changes, no evidence for reverse shoes, may need shoes with wider toebox. Rigid MTA or flexible MTA having difficulty with shoes or standing. Refer to ortho, may do stretching casts and reverse last shoe to maintain (typically around 6-12 months). Surgery is rarely indicated.

Developmental dysplasia of the hip: what is it? likelihood? diagnostic maneuvers? treatment/management?

Abnormality of the proximal femur, acetabulum, or both. Previously called congenital hip dislocation. happens to 1/1000 live births, Girls 4X more likely to be affected, Breech presentation, positive family history, Left hip > right Diagnostic: Ortolani maneuver: test of hip reduction (femoral head from dislocated positive back into acetabulum). Supine positive, knees and hips, flexed, examiner's thumb positioned on inner thigh, 3rd and 4th fingers over greater trochanter. Abduct hip, press fingers against greater trochanter and press shaft of femur toward bed with hand. Positive = palpable clunk. Barlow maneuver: test of dislocation (femoral head for within the acetabulum to dislocated position). Use same hand position, start from abducted position, adduct knee, push with thumb gentle posterior/lateral pressure on upper thigh. Positive= clunk (femoral head has slipped over lateral edge of acetabulum and has dislocated). Management & outcomes: Pavlik harness: 6-12 weeks full-time use After 12 months of age: surgical intervention, including opening the joint and cutting and realigning the femur and/or acetabulum. As the child ages: failure of good outcomes Up to 70% of children treated surgically after 3 years of age develop early osteoarthritis.

Cutis Marmorata

Accentuated physiologic response to cold. Lacy, reticulated, red or blue vascular pattern. Persistent, pronounced in trisomy 18 and 21. Usually resolves when the infant warms→ transient

RSV pathogenesis

Acute inflammatory changes --> airway obstruction and air trapping --> polyphonic wheezing, patchy atelectasis, bilateral hyperinflation. Infection begins in the nasopharyngeal epithelium, spreads rapidly by intercellular transmission through the lower airways, replication of virus is most efficient in the terminal bronchioles Sloughing of necrotic epithelial cells, exposure of dense subepithelial network of nociceptive nerve fibers, cough reflex Infiltration of peribronchiolar tissues by lymphomononuclear cells, increased microvascular permeability, submucosal edema and swelling, increased viscosity and volume of secretions, mucous plugging

Hep B: clinical manifestations

Acute phase: manifestations range from subclinical or anicteric hepatitis to icteric hepatitis, and in some cases fulminant hepatitis. Chronic phase: during the chronic phase, manifestations range from an asymptomatic carrier state to chronic hepatitis, cirrhosis, and hepatocellular carcinoma. (chronic would have elevated LFTs, but no elevated billi, so not icteric).

Measles epidemiology

Acute viral respiratory illness caused by a single stranded RNA virus. Transmitted by direct contact with infectious droplets or by airborne spread. Measles is one of the most contagious of all infectious diseases, up to 9 out of 10 susceptible persons with close contact to a measles patient will develop measles. Occurs worldwide and remains a leading cause of mortality especially among children < 5 years of age. Before the vaccine it caused over 2 million deaths annually.

Bacterial Pneumonia treatment: outpatient by age group

Age 2-3: azithromycin for 5 days or erythromycin for 14 days, consider admission. if influenza + too: oseltamavir recommended. Age 3months to 18 years: give amoxicillin 90mg/kg/day divided every 12 hours for 10 days (max daily dose 4000mg). if c. pneumonia or m. pneumonia (community acquired), then azithromycin 10mg/kg on day 1 followed by 5 mg/kg on days 2-5. if PCN allergic and not a type 1 hypersensitivity, you can use a 2nd or 3rd gen cephalosporin like cefdinir, but if a type 1 hypersensitivity, then use clindamycin.

Breast milk jaundice: what is it, cause, clinical findings, management

An elevated serum indirect bilirubin concentration in an infant receiving an adequate amount of breast milk. Peak is usually occurring after the first week of life. Cause: may be related to an enzyme that inhibits glucuronyl transferase, which causes intestinal reabsorption of bilirubin and is present in some mother's breastmilk. Clinical findings: Occurs with healthy and thriving infants with adequate stooling and voiding and appropriate weight gain. Elevated bilirubin levels typical between day 7 and 10 of life; Peaks at approximately day 10-15. May persist up to 3 months old. Different from breastfeeding jaundice which occurs at approximately day 2-7 of life and is associated with low levels of breastmilk intake. Management: Rarely does breastfeeding need to be discontinued, but transcutaneous bilirubin measurements, serum bilirubin levels, and phototherapy may be indicated.

APGAR scoring

At 1 and 5 mins from birth. activity (muscle tone): 0 points for absent, 1 point for flexed limbs, 2 points for active. pulse: 0 points for absent, 1 point for <100 bpm, 2 points >100 bpm grimace (reflex irritability): 0 points for floppy, 1 point for minimal response to stim, 2 points for prompt response to stim. appearance (skin color): 0 points for blue/pale, 1 point for pink body/blue extremities, 2 points for pink. respiration: 0 points for absent, 1 point for slow and irregular, 2 points for vigorous cry.

alopecia areata

Autoimmune disorder that causes the affected hair follicles to be mistakenly attacked by a person's own immune system; usually begins with one or more small, round, smooth bald patches on the scalp. (hard to treat). Looks like: bald, shiny, larger patches, very clean looking, no scale, no erythema, may have a few broken off hair at the margins which pull out easily.

Pediatric asthma anticipatory guidance

Avoid irritants - keep windows/doors closed, use AC, eliminate/ reduce allergen, consider allergen immunotherapy if needed Treat underlying conditions like rhinitis, sinusitis, GERD. consider additional pharmacotherapies - anticholinergics reduce vagal tone in airways -Cromolyn sodium- inhibits mast cell release of histamine -Leukotreine receptor agonist - disrupt synthesis or function of leukotrienes -Pulmonary may consider omalizumab (recombinant humanized IgG monoclonal antibody to help bind to IgE) -Exercise induced asthma: warm up for 5-10 mins, give SABA 5-20 mins prior to exercise, scarf/mask during cold weather (High risk: distance running, high endurance, track/field. Low risk: short duration, golf, volleyball, gymnastics, baseball, wrestling, football, short term track/field)

Infantile hemangioma

Blanchable erythema. Superficial or deep. Majority not seen at birth but present in the first few days. Sometimes have to US depending on size location, consult opthy if involving eye. For deep hemangiomas assess for nerve entanglement

if infant is <28 days and rectal temp >38, what is the diagnostic workup?

Blood cultures, CBC with diff, CRP. urinalysis with urine culture (catheterized specimen). consider CXR if tachypnea, retractions, cough, auscultatory findings. consider COVID, RSV, and influenza swabs. Serum PCR for HSV. Lumbar puncture: CSF cell count, protein, glucose, gram stain and culture, enterovirus and herpes PCR, herpes culture.

Getting a good latch

Both the mother and the baby should be comfortable. The infant should be positioned "face on" at nipple height so that no head turning, or tilting is required. Infant should be lying belly to belly with mother. The infant's body should be in good alignment, with a straight line from the ear to the shoulder to the hips. The infant's top and bottom lips should be flanged out, with the infant's tongue extending forward over the lower gum line and cup around the nipple and areola. Good latch-on results in quiet feedings. No sounds should be heard. After mother's milk is in, audible swallowing or air blowing out the baby's nose, should be heard.

Nipple preference or confusion: what is it, management, and complications

Breastfeeding and bottle feeding require different oral-motor skills of the infant. Infants who are given artificial nipples may attempt to breastfeed using the same sucking pattern and causing difficulty latching, fussiness, and breast refusal. Management: Avoid bottles or pacifiers for the first 4-6 weeks or until infant is breastfeeding well. Use syringe, cup, or supplemental nutrition system when supplementation is needed. Consult with lactation specialist to retrain infant to have effective suck & proper latch. Use of nipple shield to retrain infant. Complications: Failure to thrive, hyperbilirubinemia, colic and crying, prolonged feedings, sore and cracked nipples, plugged milk ducks, mastitis, and frustration if ongoing problem.

For human milk storage guidelines, which statement is NOT true?

Breastmilk leftover from a feeding has to be used within 4 hours

diagnostic evaluation for pediatric asthma

CBC and Differential: if concern for infection or anemia (will also give information about eosinophils) CXR not indicated as routine. Recommended with persistent wheezing without clinical explanation, hypoxia/fever/ suspected pneumonia, localized rales requiring admission. consider allergic evaluation Sweat test if continuing to get sick to rule out CF Pulmonary function tests: Spirometry = gold standard, perform on a regular basis to evaluate/manage/monitor, can usually be performed in kids > 5 yrs Peak flow measurements: kids 4-5 years old Evaluated based on child's personal best and monitored over time personal best obtained by taking measurements twice daily (morning, late afternoon) 15-20 mins after SABA Green zone - 80-100% of personal best Yellow - 50-79% Red - 0-49% Exhaled nitric oxide: moderate accuracy with kids > 5 yrs, more accurate when not on steroids, measures eosinophilic airway inflammation

well appearing 3 to 36 months, and immunizations not complete with fever. what treatment?

CBC with diff, blood culture if WBC >15,000. UA with urine culture, CXR if indicated, treat with cefriaxone 50mg/kg IM pending cultures if abc >15000. FU in 24 hours.

Neonatal staph pyoderma MSSA

Can look like bullae (fluid filled pockets, some ruptured some not. Give IV antibiotics and admit

Common breastfeeding challenges: flat or inverted nipples (pinch test)

Can make it more difficult for infant to pull nipple in mouth for sucking and achieve a good latch. Inverted nipples: If infant is able to latch, the nipple will elongate with time and problem usually become less severe. Flat Nipples: Often found in women with larger, pendulous breasts. Flat nipples do not usually change over time. Infant may be able to develop a style to more easily latch on successfully Pinch test: If the nipple pulls in, it is inverted. If the nipple does not pull in or everts with compression, it is flat

anticipatory guidance for toddlers and preschoolers

Car seat - both the actual car seat and the parent modeling appropriate behavior by always wearing a seat belt Falls - stair gates, window guards, keep furniture away from windows Close supervision - especially around cars, streets, machinery Choking prevention - supervision while eating, cutting up food into small pieces

Presenting illness/ signs and symptoms of Kawasaki disease, treatment

Case 1: previously healthy male, recently moved from CA with fever for 6 days, rash for 2 days. Decreased oral intake, decreased urine output for 3 days, fussy for past 6 days. ROS + for runny nose, runny eyes, no vomiting or diarrhea. PE: Febrile, baby looks sick and is intermittently fussy and apathetic. TMs red, dull, bulging bilaterally. No lymphadenopathy (could have one isolated large node). Chest clear, heart normal RRR. Abdomen soft and nontender. Eye injected with minimal discharge, mild nasal discharge, limbic sparing. Lips injected and fissuring, strawberry tongue, throat red, no tonsillar exudates. edema of dorsum of hand and feet. Rash: generalized maculopapular erythematous rash, confluent in some areas. Diaper area: erythema, crusting, and desquamation. diagnostic: diagnosis of exclusion Tx: IVIG, aspirin for anti-platelet effects.

Presenting illness/ signs and symptoms of Tinea capitus

Case 2: 6 week history or scalp rash, getting worse, no known contacts with similar rash. hair loss, itching, and pain increasing. 1 day history or pus running out of rash. fever/cough/runny nose for 2 days. seen by PCP one month prior, treated with griseofulvin V suspension for 1 month and atarax for itching. mother also washing childs hair TID with selenium sulfide shampoo. PE: alert, well appearing, low grade fever. Scalp: boggy erythematous rash with purulent drainage. occipital nodes enlarged, dry cough, mild coryza, chest clear.

Rocky Mountain Spotted Fever (RMSF): etiology, S/S, diagnostic, Treatment

Case3 HPI: multiple episodes of vomiting and abdominal pain over the last 3 days. fever to 39.3 for last 3 days, muscle joints and ache. 48 hours of rash starting on his legs, progressing to his entire body but more extensive on his arms and legs. Pt vacationing with family in oregon when became ill. Etiology: tick born bacteria rickettsia, distributed nation wide. Signs/symptoms: fever, headache, muscle pain, petechial rash on day 2 to 5 of fever starts peripherally, progresses centrally. can lead to life threatening disease with vasculitis leading to heart, lung, kidney, and CNS involvement. Diagnostic: low platelets, low NA, elevated liver enzymes, antibody titers, immunofluorescence. Skin biopsy: don't wait for the results to treat! Treatment: docycycline!

Stages of Pertussis

Catarrhal: 1-2 weeks. nonspecific complaints, URI/common cold-like, worsening cough, coryza, sneezing, low-grade fever. paroxysmal: 2-4 weeks. afebrile to minimal fever, persistent staccato cough, cough ends with inspiratory whoop, post-tussive emesis, cyanosis, sweating, exhaustion, disturbed sleep. *the cough that manifests is an attempt to dislodge necrotic bronchial epithelial tissue and thick mucus plugs. Convalescent: 3 weeks - 6 months. symptoms eventually wane over months. *symptoms vary by age, stage in the disease process, and immunization status. findings are especially severe in infants <6months old. in 50% of adolescents, the classic cough can last longer than 10 weeks. infants may have a cough without the whoop. their catarrhal phase can be relatively short, but their recover phase is prolonged.

Sore nipples: cause, mangement, complications

Cause: improper latch or positioning, prolonged negative pressure, inappropriate suction release from the nipple, sensitivity to nipple creams or oils, incorrect use of breast equipment (pumps, shields), thrush, and leaking nipples that are not properly air-dried. Management: Prevention by early assessment and education on latch and position; Referral to lactation specialist. Express a few drops of colostrum onto the nipple and let air-dry after feedings. Nurse from least sore breast first or pump from the affected breast if too severe to nurse. Use mild analgesic. Complications: Breakdown of breast skin integrity and increased risk for infection. Interference with breastfeeding

Hemorrhagic disease of the newborn: what is it? early vs. classical vs. late? management?

Caused by lack of vitamin K in mother and lack of vitamin K production in infant due to absence of bacterial producing intestinal flora Early: 0-24 hours after birth. Severe, mainly found in infants whose mothers used certain meds that interfere with how the body uses vitamin K. Classical: 1-7 days after birth. Bruising, bleeding from the umbilical cord Late VKDB is rarer, occurring in 1 in 14,000 to 1 in 25,000 infants (1-3). 2-12 weeks after birth is typical, but can occur up to 6 months of age in previously healthy infants. 30-60% of infants have bleeding within the brain, tends to occur in breastfed only babies who have not received the vitamin K shot, warning bleeds are rare. Infants who do not receive a vitamin K shot at birth are 81 times more likely to develop late VKDB than infants who do receive a vitamin K shot at birth. (4) Management: 1 to 5 mg of Vitamin K IV, cessation of bleeding should occur within a few hours. Prevention by routine administration of 1 mg of natural oil-soluble vitamin K IM within 1 hour of birth.

Varicella Epidemiology, manifest, and complications

Caused by varicella-zoster virus (VZV) peaks in march-may, marked decreased since vaccination (1st dose is 85% effective, and 99% with 2nd dose). Manifests: diffuse generalized pruritus vesicular rash. progresses rapidly from macule to papule to vesicular lesions before crusting. usually appears first on head, chest, and back then spreads to the rest of the body. "breakthrough rash" is less severe Complications: bacterial SSTI

severe engorgement: cause, management, complication

Characterized by fullness, soreness, and swelling of both breasts as the milk transitions in volume. Caused by milk stasis in the breast from inadequate emptying. Management: Prevent with frequent nursing with good latch and avoid long stretches without feedings. Use warm compresses or hot shower for 5-10 minutes before nursing. Gentle massage prior to pumping or nursing. Manually express before feeding to soften the areola & make it an easier latch Complications: My become painful, warm, and lead to mastitis.Nipples may flatten leading to difficulty latching.

Developmental red flags for requiring immediate referral

Children who stop eating, children who demonstrate cruelty to animals or people, children who are self-harmful, children who start fires, children who talk of harming themselves, their peers or others

Fever of unknown origin (FUO)

Children's fever >101 of at least 8 days, in whom no diagnosis is apparent after initial output or hospital evaluation.

valley fever clinical findings

Clinical findings: flulike disease with cough and fever, some develop a skin reaction following several weeks of illness. •Typical history: fatigue, fever, weight loss, rash on upper body or legs, cough and headache, malaise, chest pain (with pulm infection), myalgia or joint pain, and neck stiffness (with meningitis) •Physical exam: fever, crackles (with resp infection), and cutaneous abnormalities including erythema multiforme and erythema nodosum.

Epiglottitis: clinical findings, organisms, management, complications, prevention

Clinical findings: usually in children 1-5 years old. Rapid onset, associated with sore throat dysphagia, anxiety, respiratory distress, no significant stridor, drooling, muffled speech, toxic appearance, tripod position, high fevers. "cherry red" and enlarged epiglottis. elevated WBC with left shift. (wouldn't necessarily need an xray but would show thumb's sign). Course is usually 48-72 hours. organisms: GAS, strep pneumonia, klebsiella sp., staph aureus. Hib if not vaccinated. Management: ◦This is an emergency! ◦Do not intervene, notify anesthesia/OR of case and prepare for intubation in the OR. ◦Obtain cultures during intubation ◦Initiate IV broad spectrum antibiotics - amp/sulbactam, cefotaxime, ceftriaxone, or clinda if PCN allergic. Vanco if MRSA suspected. 10-days of antibiotic therapy ◦Initiate corticosteroids complications: spread of infection, routine immunization provides primary prevention prophylaxis of contacts: if Hib identified as causative agent, give Rifampin prophylaxis to household contacts if: At least one child < 4yrs who is un/incompletely immunized or Children < 12 months who have not completed series, or Immunocompromised status

posterior fontanel closure

Closes by 2 months of age (between 6 and 8 weeks)

How to prevent the spread of pertussis? what to give family members

Cocooning: protect infants from dangerous environmental hazards Universal immunization of children with DTaP in initial series. Target adult immunization: caretakers, grandparents over 65 yrs, fathers, mothers who did not get immunized in the last trimester, Tdap for adolescents 11-18 years Macrolide for household and close contacts irrespective of immunization status. Considered anyone within 3-4 feet of symptomatic person, shared closed space for 1 hour, or direct contact with secretions Chemoprophylaxis: ◦Household and close contacts ◦Value limited after 21 days of illness, but should be considered in high-risk individuals: Review and update immunization status, Monitor school contacts for symptoms for 21 days, Exclude symptomatic students and evaluate everyone with a cough

Caput cephalohematoma

Collection of blood in the subperiosteal area (between the skull and periosteum) of the scalp that does not cross the suture lines. Bruising is often not seen. It appears hours to days after delivery. No treatment, disappears on its own.

Molluscum contagiosum

Color: white, waxy appearance: papules Distribution: grouped with satellites Symptoms: none Pruritic: no

Breast milk jaundice: benign unconjugated hyperbilirubinemia

Common cause of prolonged jaundice in healthy term infant. Bilirubin due to BF does not exceed 12mg/dl. Presents in the first 1 to 2 weeks. May persist for up to 12 weeks. Incidence in exclusively breast fed infants→ 36%.

Foreign body aspiration: laryngeal, tracheal, and bronchial

Common in toddlers, may have acute or gradual symptoms. general S/S: minor to significant distress depending on degree of obstruction. Laryngeal: hx of rapid onset of hoarseness, croupy cough, recurrent pneumonia. PE: unilateral wheezing and sudden episode of cough on. eval: expiratory/lateral xray, direct laryngoscopy Tracheal: hx of brassy cough, hoarseness, dyspnea, cyanosis. PE: monophonic wheeze, audible slap/palpable thud at subglottic level secondary to expiratory effect. Bronchial: hx of coughing, gagging, choking episode. PE: similar to other aspirations, diminished breath sounds, decreased vocal fremitus, limited chest expansion hyperresonance. eval: inspiratory and forced expiratory chest X-rays, chest fluoroscopy Xray findings with foreign bodies: may appreciated localized hyperinflation, mediastinal shift, atelectasis, object present

Cardiovascular complications of kawasaki disease

Coronary artery aneurysms Cardiomyopathy with depressed myocardial contractility and heart failure Myocardial infarction, Arrhythmias Peripheral arterial occlusion

Positions for breastfeeding: Cradle, cross-cradle, football hold

Cradle: baby's head is supported in the crook of the elbow on the same side as the breast being suckled Cross-cradle: opposite hand supports the baby's head and shoulders Football hold: •Infant is supported off the side of the mother Pillows should be placed at the mother's side to help support the infant. Baby is in a side-lying position, flexed hips, with buttocks back against chair. Position is used with newly breastfeeding mothers or mothers with c-sections

Storage of thawed, previously frozen

Countertop (room temperature): 1-2 hours. refrigerator: up to 1 day. Freezer: never refreeze after it has been thawed

Storage of freshly expressed or pumped

Countertop (room temperature): up to 4 hours. refrigerator: up to 4 days. Freezer: within 6 months is best, up to 12 months is acceptable.

Hand foot mouth disease: etiology, S/S, diagostic, Treatment

Coxsackie A 16, 6, and other enteroviruses. incubation period of 4 to 6 days. highly contagious in children. more common in the summer S/S: grouped coarse papules or vesicles on buttocks, hands, knees, and elbows (usually extensor surface). Lesions on tongue and buccal mucosa. not directing or causing cellulitis. Hands usually > feet

Hand, Foot, and Mouth Disease (HFMD)

Coxsackie A. Fever, vesicular eruption on buccal mucosa, maculopapular rash on hands and feet. lasts 1 to 2 weeks.

Herpangina

Coxsackie A. sudden onset fevers up to 106 lasting 1-4 days. anorexia, sore throat, dysphagia, painful. Minute vesicles: 1 to 15 lesions of 1-2 mm each. Enlarge to ulcers on the posterior pharynx. Entire course lasts 3 to 7 days with complete recovery.

diphtheria diagnostic studies and management

Diagnostic studies: Confirmatory culture positive of C. diphtheria (specimen of nose, throat, or any skin lesions) Management: children with diphtheria require early tracheostomy placement for airway stabilization. Treatment: Anitoxin: a single dose given if there is a high suspicion prior to culture result. Antimicrobial therapy: erythromycin given orally or IV for 14 days. (or Penicillin G for 14 days either IM or IV) Supportive care for resp, cardiac, and neuro complications Droplet precautions until 2 negative cultures. Immunize after recovery

step 3: moderate persistent asthma symptoms and lung function

Daily symptoms, daily use of SABA, exacerbations affect activity > or = 2x/week, may last days. nighttime symptoms more than once per week, but not every night lung function: FEV 60-80% of predicted

Mongolian spots

Darker pigmentation, looks like bruising. Common areas: buttocks, lower back, knees, shoulders. Important to document incase of child abuse issue later. No treatment

valley fever diagnostic studies and management

Diagnostic studies: enzyme immunoassay (EIA) tests assess IgM and IgG. Coccidiodioides can be seen under microscope Management: Outpatient: fluconazole or itraconazole for 1 year minimum. Inpatient/Severe: admission and IV amphoB or intrathecal amphoB *patients with CNS disease may require lifelong therapy.

DTaP vs. Tdap

Diphtheria + Tetanus + acellular Pertussis = DTaP for kids < 7 years old Tdap is the BOOSTER given to children > or equal to 7 years Td: adults who have received a minimum of 1 Tdap vaccine.

Incomplete (atypical) kawasaki disease

Does not fulfill four criteria with fever > 5 days: Adenopathy <90% Rash < 50% Peripheral extremity changes absent 40% Mucous membrane changes were most characteristic of KD and were present in more than 90 percent of children with either typical or incomplete disease

Inborn errors of metabolism: what are they? how to we test for them?

Due to the absence of abnormality of an enzyme or its cofactor, leading to either accumulation or deficiency of a specific metabolite. Early diagnosis key to well being of an infant. A delay in diagnosis may result in acute metabolic decompensation, progressive neurologic injury, or death. Most are inherited as autosomal recessive traits. Evaluation: michigan tests for 56+ disorders plus hearing/CCHD. Elevated plasma ammonia levels, hypoglycemia, and metabolic acidosis are suggestive of inborn errors of metabolism. Unusual odor (maple syrup disease, PKU, hepatorental type 1)

explanation of biphasic asthma response

Early asthmatic response (EAR): Activation of mast cells and mediators, begins with bronchoconstriction about 15-30 minutes after activation. Resolves within one hour if allergen is removed. Typically responds well to inhaled bronchodilators Late-phase response: Prolonged inflammatory state, usually follows EAR. Occurs within 6-26 hours of exposure. Associated with more severe airway hyperresponsiveness that may last hours to weeks •Requires corticosterd

Herpes simplex 1 gingivostomatitis etiology, S/S, diagostic, Treatment

Etiology: HSV1 primary infection S/S: high fever, severe mouth pain. tongue, buccal mucosa, gums, tonsils, lips and skin around mouth inflamed. vesicles form and rupture into ulcers. Foul odor, bleeding and fissuring of lips. submental adenopathy. May inoculate to fingers because kids put hands in their mouths (herpetic whitlow). Treatment: acyclovir may shorten the course if started early.

Meningococcemia etiology, S/S, diagostic, Treatment

Etiology: Neiseria meningitidis, gram negative diplococcus. S/S: spectrum from fever and occult bacteremia to sepsis, shock, and death. Purpuric and petechial rash. **may be rapidly progressive over a period of hours. Diagnostic: positive cultures TX: IV antibiotics start even when cultures are pending

Scarlet fever etiology, symptoms, diagnostic, and treatment

Etiology: an exotoxin produced by strep progenies (GAS) symptoms: sore throat, fever, strawberry tongue, sandpaper rash, red flush, systemic symptoms, peeling. Rheumatic fever: glomerulonephritis Diagnosis: rapid strep, throat culture, ASOT, elevated WBC with left shift. Treatment: amoxicillin BID, penicillin TID, IM benzathene penicillin, keflex, other cephalosporins, azithromycin **be careful, Kawasaki disease and staph exotoxin reaction can look identical

What's the name of the pediatric asthma set of comprehensive guidelines?

Expert panel report 3 (EPR-3). Guidelines for the Diagnosis and Management of Asthma ◦Best available scientific knowledge to clinical practice ◦Published in 2007 ◦Builds of asthma guidelines issues in 1991, 1997 and update release in 2002. ◦Consists of a full report and a summary report

Spirometry FVC vs FEV1

FVC: forced vital capacity Interpretation of % predicted: ◦80-120% Normal ◦70-79% Mild reduction ◦50%-69% Moderate reduction ◦<50% Severe reduction FEV1: forced expiratory volume in 1 second. interpretation of % predicted: >75% normal, 60-75% mild obstruction, 50-59% mod obstruction, <49% severe obstruction (if <25 years old at 5% and >60 years subtract 5%)

Kawaski disease diagnostic criteria

Fever > 5 days duration with 4 out of 5 of the following: Bilateral bulbar conjunctivitis. Oral mucous membrane changes including injected or fissured lips, injected pharynx or strawberry tongue. Peripheral extremity changes including erythema of palms or soles, edema of hands or feet (acute phase) or periungual desquamation (convalescent phase) Polymorphous rash Cervical lymphadenopathy -at least one lymph node >1.5cm in diameter

About fever

Fever is the most common pediatric complaint, second only to routine care for clinic visits, and the most common reason children are brought to the emergency department (10-25 %) 5-20% have no localizing signs on PE with no history to explain the fever. The majority of children with fever do not have a serious illness, although a small percentage harbor or may develop a serious bacterial infection (SBI). The responsibility of the clinician is to identify those at risk for SBI

signs and symptoms of infection in a neonate

Fever or hypothermia. Tachycardia, bradycardia, tachypnea, hypotension. Lethargy or irritability (not fussiness) Poor feeding or vomiting Diarrhea or abdominal distension Apnea Dyspnea or low oxygen saturations Jaundice Bulgingfontanelle Seizures Rash(vesicular)

Milia

Firm, pearly white papules over cheeks, nose, and forehead. Common, benign, keratin filled cysts, self resolves→ no treatment indicated. Oral counterparts are called epstein pearls.

Successful breastfeeding urine output for days 2-4

First 2 days: 1-3 diapers in 24 hours Day 3: 3 or more diapers in 24 hours Day 4: 4 to 6 diapers in 24 hours Overtime, infant should have 6 to 8 diapers in 24 hours. Parents keep diary of wet diapers

Neonatal HSV 1 and 2: S/S, diagostic, Treatment

First 6 weeks of life contracted in utero or during delivery. Presentation may be rash of grouped vesicles, fever without rash, seizures, jaundice, or other nonspecific signs of sepsis. 35% have encephalitis. Diagnostic: HSV, PCR, and viral cultures of skin, CSF, mouth and eyes. MUST do an LP. MRI of the brain. TX: IV acyclovir, management of sequelae of sepsis and encephalitis.

12-18 months Language

Follows one step commands, uses all vowels, increased use of real words, likes to use negatives (NO), understands simple questions

18-24 months language

Follows two step commands without visual cues, dramatic increase in vocabulary (200+words), names self, enjoys simple stories and songs

valley fever cause and incubation

Fungal infection caused by C. immitis and C. posadasii in southwestern US, Mexico, central America, and south america Incubation:7-28 days. Person to person transmission only occurs in congenital infection or in pts with draining skin lesions

Streptococcal disease

Gram positive spherical cocci. Classified based on ability to hemolyze RBCs: complete, partial, and no hemolysis Group A beta hemolytic strep (GABHS): most virulent Group B strep (GBBHS) causes bacteremia and meningitis in infants <3 months of age. Non-group A or B: associated with invasive disease in any group

diagnosis of asthma: guided by? delay until how old?

Guided by the National Asthma Education and Prevention Program Expert Panel Report (EPR3) Provides classifications of asthma, recommendation for stepwise approach to pharmacologic intervention, diagnosis Often delayed until at least 4 years old Initially diagnosed with reactive airway disease, wheezy bronchitis, recurrent bronchitis Rarely delayed before 12 months because of the number of viral illnesses that cause bronchiolitis Environmental allergens - worsened by global warming, climate change, air pollution house dust mites, cockroaches, indoor molds, Saliva/dander of dogs/cats Outdoor seasonal molds Airborne pollens - trees, grasses, weeds Food allergies (tree nuts, eggs) Comorbid conditions: sinusitis, gastroesophageal reflux

why has occult bacteremia in fever without source dropped dramatically in 3 mo to 3 years age group?

HIB vaccine in 1991 and pneumonococcal vaccine in 2000 and herd immunity

specific patterns for rashes

Hand foot mouth (self explanatory) Dermatomes (herpes zoster) Grouped (herpes, molluscum contagiousum) Rings (ringworm, iris or target lesions)

discharge criteria

Healthy prior to fever, no risk factors child appears non-toxic and healthy. caregivers reliable and have access to transportation. lab parameters are not critical.

herpes labialis and herpetic whitlow

Herpes labialis (cold sore or fever blister) limited to vermillion border Herpetic whitlow or viral paronychia is a painful localized infection of the digit. Occurs in children who suck their thumbs, bite nails, have gingivostomatitis

Treatment for persistent asthma: step 5

High dose ICS + either a LABA or montelukast consult with specialist

Measles clinical manifestations

Highly contagious viral illness characterized by fever (as high as 105), malaise, rash, cough, coryza, and conjunctivitis. (the 3 Cs). Koplik spots followed by a maculopapular rash. rash spreads from the head to the trunk to the lower extremities.

diphtheria HPI and manifestations

History: low grade fever and gradual onset of symptoms over 1 to 2 days with bacterial shedding for 2-6 weeks if untreated. Fully immunized individuals can carry the bacteria asymptomatically and may present with just a mild sore throat. Physical exam: Hallmark is thick, grayish adherent pseudo membrane found in either the nasopharynx, pharynx, or trachea that bleeds on removal., Bloody nasal discharge, sore throat, serosanguineous nasal discharge, hoarseness, cough. Extensive neck swelling with cervical adenitis (bull neck) characterizes severe disease and causes airway obstruction. Cutaneous lesions (nonhealing ulcers with dirty gray membrane), possible otic and or conjunctival infection findings. Clinical indications of toxin production: myocarditis and ECG changes, resp compromise, cranial nerve and local neuropathies, and peripheral neuritis.

Croup: management, ongoing evaluation, when to hospitalize

Humidified air: not proven helpful or hurtful in mild to moderate croup. Cold air can be helpful. Taking children for a car ride on a cool night with the windows down can help. Alternatively, place by an open freezer. No use of steam. Nebulized epinephrine may help, but will require the use of simultaneous corticosteroids to help limit rebound swelling Corticosteroids: decreases inflammatory edema, prevents destruction of ciliated epithelium. Dexamethasone (oral or IM 0.6mg/kg). Limit use to 1-2 days. If admitted, dexamethasone 0.5-2mg/kg/dose every 8 hours may be necessary. No antibiotics. No cold medicines. oxygen only if hypoxic, bronchodilators if bronchospasm, heliox only in severe cases. Ongoing evaluation: 'Red Flags: Increased retractions, air hunger, restlessness, hypoxia, weakness, lethargy, decreased air exchange, decreased stridor, increased tachycardia, eventual death from hypoventilation Use caution with tongue depressor: may result in sudden compromise hospitalize: ◦Significant distress (RR 70-90) ◦Stridor at rest ◦Temp > 39oC (102.2oF) ◦Inability to tolerate adequate PO intake

Staph scalded skin treatment

IV dicloxacillin, a 1st or second generation cephalosporin, or clindamycin. as well as a topical mupirocin at the infection site.

infants 1 to 3 months: how to follow-ups with low risk infants?

If all cultures negative: afebrile, well appearing→Careful observation Blood cultures negative: well appearing, febrile→Careful observation, may consider second dose of Ceftriaxone Blood culture positive→admit for sepsis workup and parenteral antibiotics pending results Urine culture positive: if persistent fever→admit for sepsis workup, parenteral antibiotics pending results. If afebrile and well→outpatient antibiotics

MRSA treatment

If fluctuant lesion, incision and drainage. First line anti-biotic therapy: Trimethoprim-Sulfamethoxazole (Bactrim) (Warning: Steven-Johnson syndrome, toxic epidermal necrolysis, Not active against S. pyogenes, Could add cephalexin) Clindamyacin: Diarrhea, rash, pseudomenbranous colits, Steven- Johnson syndrome. Clostridium difficile-induced colitis (D test- if area around the clindamycin disc is D shape- test is positive indicating possibility of resistant pathogens)

Urticaria epidemiology, S/S, treatment

Immune response/ hypersensitivity reaction Raised flat wheals which move around, very itchy, comes and goes. sometimes associated with puffiness of lips, ears, eyelids, genitalia. Key finding: appears suddenly, fades 20 mins to 24 hours, blanch with pressure Tx: quick resolution, identify and remove offending agent, give oral antihistamines, topical antipruritics, epi or prednisone if anaphylactic or angio edema

breastfeeding contraindications

Infants with certain metabolic disorders such as galactosemia or PKU. Maternal diagnoses: human T-cell lymphotropic virus type I or II, maternal cancer, untreated brucellosis, HIV infection,

Mastitis: what is it? predisposing factors? management and complications

Infection of the breast during lactation causing malaise, breast tenderness, a reddened, warm lump sometimes associated with red streaking, and flulike symptoms including fever, chills, and body aches. Predisposing factors: stress, fatigue, cracked nipples, blocked milk ducts, constricting bra, inadequate emptying of the breast and sudden weaning or decrease in number of feedings. Management: Warm compresses to allow breast to empty and avoid engorgement. Continue with frequent feedings. Attending to cracked or sore nipples immediately. Rest, increase fluids, and use analgesics as needed. Antibiotic treatment recommended when conservative & supportive care is ineffective. DO NOT wean abruptly Complications:Abscess and septicemia

bronchiolitis: what is it? what does it look like? what causes it?

Inflammation, necrosis, and edema of respiratory epithelial cells in the lining of the small airways + copious mucus production. gradual onset of URI symptoms. progresses to lower respiratory symptoms lasting up to 12 days. severe disease most common at 2-3 months because they aren't seeing maternal immunoglobulins anymore. Most common respiratory infection in infancy (10% of primary care visits in the first 2 years of life >80% of bronchiolitis occur in infants < 1 year old) S/S: Along with onset of URI symptoms, decreased feeding and mild fever, apnea in younger infants. Wheezing and crackles usually appear with tachypnea after 1-2 days, mild to severe chest retractions. Causes: RSV, parainfluenza, adenovirus, human metapneumovirus

Scabies: etiology, clinical findings, treatment

Insect infestation of sarcopti scabeii. transferred person to person. not all contagious victims are symptomatic. It is an immune reaction to the insect. key finding: itching worse at night, complaints are worse than physical findings. fitful sleep, curving burrows especially in the webs of fingers, sides of hands, folds of wrist, armpits, forearms, elbows. treatment: treat with permethrin 5%, repeat in 1 week. use antihistamine, hydrocortisone, or NSAIDs for itching. treat everyone in the family and at daycare also even if asymptomatic.

Newborn cord care

Leaving the cord to air dry and placing the diaper below the cord until it separates around 10-14 days old. Alcohol application is no longer recommended. If granuloma appears after the cord falls off, an application of silver nitrate helps to heal it.

AAP for media use/screen time for toddlers and preschoolers

Limiting media time to no more than 1 hour per day for children 2-5 is recommended so that children have plenty of time for other activities to promote development <18 months = discourage screen media other than video-chatting 18-24 months = high quality programming/apps >2 years = 1 hour per day or less of high-quality programming (Use with an adult) 2-5 years = no more than 1 hour per day Screen free times:During meals, 1 hour before bed, Parent-child playtimes, Screen free "zones" Ex. screen free zone = the child's bedroom. This discourages the child from using the media for extended periods of time, and without supervision. The media should be collected by the parent before the child goes to bed.

30-36 months language

Listens to adult conversations, can categorize items by function, begins to recognize color, three- to four- word sentences, can help tell a simple story

pertussis treatment

Macrolide antibiotics Erythromycin Dosing: infants 1-5months: 10mg/kg/dose four times daily for 14 days. infants 6months and children: 10mg/kg/dose four times daily for 7-14 days. adolescents: 500mg four times daily for 7-14 days. Azithromycin *drug of choice for infants < 1 month. Dosing: ◦Infants 1-6 months of age: 10mg/kg/day as a single dose once daily for five days ◦Infants >6 months, children, adolescents: 10mg/kg/day as a single dose on day 1, followed by 5mg/kg/day as a single dose on days 2-5 ◦Loading max dose 500 mg, daily max dose 250 mg Clarithromycin ◦Used in infants > 1 month, children, and adolescents Dosing: ◦15mg/kg/day in two divided doses for 7 days (max dose 1g/day) Sulfamethoxazole/trimethoprim: alternative for macrolide-intolerant patients >2 months of age. Dosing: TMP 7mg/kg/day and SMX 40mg/kg/day in 2 divided doses (max TMP single dose 160mg) When antimicrobials are given in the paroxysmal stage, the course of illness is not altered, but the spreading of the organism is. Corticosteroids and beta-2 adrenergic med us are not supported

Poor weight gain management and complications

Management: Obtain complete history regarding infant and maternal factors. Assess breastfeeding technique to determine feeding problems. Provide instruction, encouragement, reassurance and possibly lactation consultation for correct breastfeeding technique. Supplemental nursing system at the breast if supplementation with expressed breast milk or formula is indicated. May be need for hospitalization if infant has lost too much weight, is not able to show vigor at the breast, or is showing other signs of dehydration. Refer for treatment of physical or organic causes. Complications: DD, poor bonding, severe dehydration

Meningococcal vaccines

MenACWY: in children > 11 years, these serotypes will cause 75% of disease. Serogroup B (menB): recommended for younger children in high risk categories like asplenia or complement disorders (also teens especially if going to college)

Neonatal fever most common reasons

Meningitis should always be considered—up to 10% appear well, only 15% have a bulging fontanelle, and 10-15% have nuchal rigidity. So, a high index of suspicion is important! ~20% of infants with serious bacterial or viral infections will not have fever initially. Hypothermia (< 35.O C rectal) is a more ominous sign than fever. The majority of febrile neonates have a nonspecific viral illness 12% have serious bacterial infections (SBI) (this figure varies by site and time of research). GBBS is associated with high rates of meningitis(39%), non-meningeal foci(10%), and sepsis(7%) The most common bacterial infections are UTI and occult bacteremia

community acquired MRSA

Methicillin-resistant Staphylococcus Aureus (MRSA) is resistant to class of antibiotics called the anti-staphylococcal penicillins. Majority of cases are uncomplicated SSTI Other sites: Pediatric retropharyngeal abscess, Deep neck abscess, Necrotizing MRSA complicating bacteremia or viral respiratory infection, Concurrent infection with influenza especially H1N1. increased risk in children, recent hospitalization, recent antibiotic therapy, chronic skin disease (eczema), affected family member

Infant weight gain

Normal newborn infants lose about 5%-10% of their birth weight in the first few days of life. Once maternal breastmilk volume increases, infants gain 0.5-1oz/day or 4-7oz per/week. Many breastfed infants regain their birth weight by 2 weeks. Others may take up to 3-4 weeks. After the first 24 hours, infant should be going to breast 8 to 12 times (q 2-3 hours) in 24 hours for about 20- 45 minutes at each feeding. Breastfed infants double their weight at 4-6 months and triple their weight by 1 year.

Kawaski disease: epidemiology and findings

One of the most common vasculitides or childhood with widespread inflammation of primarily medium-sized muscular arteries. usually self-limited, fever lasting an average of 12 days if not treated. etiology unknown. Diagnosis is based upon evidence of systemic inflammation (eg, fever) in association with signs of mucocutaneous inflammation. These findings are often not present at the same time, and there is no typical order of appearance. Consider if unexplained fever > 5 days. If >4 are present may make diagnosis on day 4 of illness

Croup: etiology, organisms, findings

Organisms:human parainfluenza type 3, influenza A&B, human coronavirus, coxsackie, echovirus, metapneumovirus, adenovirus, rhino. etiology and epidemiology: croup is most common for 6-36 months. human parainfluenza virus 3 is endemic in <6 months. incubation period is 2-4 days. more common in males than female. duration of illness: 5 days clinical findings: ◦URI prodrome ◦Hoarse, barking cough (brassy, harsh sounding) ◦Inspiratory stridor with dyspnea ◦Onset of symptoms over 2-3 days, worse @ night ◦+/- sore throat ◦Dyspnea, tachypnea, retractions (substernal, intercostal) ◦Low-grade fever (but may be elevated to 40C) ◦Normal epiglottis ◦Prolonged inspiratory phase extrathoracic process. make diagnosis with clinical signs, but if you were to get an xray, you'd see steeple sign.

bronchiolitis prevention

Passive prophylaxis: palivizumab (synagis) IgG monoclonal antibody administered monthly (IM injection) x's 5 injections (maximum) in the first year after birth for high-risk infants such as heart disease, lung disease of <32 week prematurity no palivizimab in otherwise healthy infants with a GA of at least 29 weeks. WASH HANDS, use of masks in contacts

What things do you ask about when getting a complete history for respiratory complaints?

Past medical history? History of hospitalizations, surgeries, chronic health issues? Sick contacts? Possible foreign body? Immunization/travel history? tB status? Other manifestations like restlessness, chills, apprehension, SOB, malaise, pleuritic chest pain? Any history of a URI or similar symptoms? Abrupt high fever? Chills? Cough? Dyspnea? (suggestive of bacterial pneumonia) Further assessment of the cough -Wet/dry? Duration? Acute (< 2weeks), subacute (2-4 weeks), recurrent, or chronic (> 3 months), Productivity? Mucous producing? Remember young children do not expectorate. Timing - daytime, nighttime, or both? Nighttime associated with asthma. Association with exercise? Associated symptoms? High fever worrisome for bacterial or viral pneumonia Allergic rhinitis/asthma: rhinorrhea, sneezing, wheezing, atopic dermatitis, pale boggy mucosa Exposure - travel history? Sick contacts?

Pertussis: epidemiology

Pathogen: bordetella pertussis (gram negative bacillus). highly contagious respiratory disease. may be primary disease of reinfection. since 1990 outbreaks are increasing despite increased vaccination rates. rate in the US has decreased incidence with prenatal vaccination of mothers.

Tetanus epidemiology:

Pathogen: clostridium tetani, found in environment (dust, soil, manure). Usually caused by a puncture wound in an unvaccinated person. in the presence of anaerobic conditions, the spores germinate producing a toxin that causes painful muscle contractions.

Diphtheria epidemiology: pathogen, clinical manifestations

Pathogen: corynebacterium, rare infection of the resp tract. no US cases since 2003. Clinical manifestations: low grade fever, grayish pseudomembrane found in nasopharynx, pharynx, or trachea. Bloody nasal discharge, sore throat, extensive neck swelling with cervical lymphadenitis.

Erythema infectious (fifth disease/ parvovirus B19): transmission, stages.

Peak age 5-15 years but all ages affected. spread vertically (mother-infant), respiratory tract secretions and blood exposure. 3 stages: Prodrome: mild fever (15-30%) myalgia, headache, malaise, URI. Rash appears 7-10 days after prodrome. First: intense redness on cheeks (slapped cheek) with circumoral pallor for 1-4 days. Second: lacy maculopapular rash on trunks moves to arms, thighs, buttocks. can last 1 month and may be pruritus. (subsides but can recur with trauma, heat, exercise, and stress).

Transient neonatal pustular melanosis

Pigmented macules, superficial pustules and vesicular pustules that rupture easily and leave a halo of white scales around. Pustular phase rarely lasts more than 2-3 days. Self resolves with no treatment indicated. Although hyperpigmentation can persist for up to 3 months

toddler and preschool stages of development: erikson and piaget

Piaget: preoperational thinking, symbolic thinking, egocentric thinking. achieving a sense of independence and autonomy. Erikson: autonomy vs. shame and doubt (will) Initiative vs. guilt (purpose)

Red flags for cardiac disease in a newborn

Positive family history Usually term AGA infant SpO2 < 95% Dysmorphic features or suspected syndrome Prominent heart murmur Absent femoral pulse Abnormal ECG or CXR

Best view to order for a chest X-ray?

Posterioanterior (PA) and lateral

Terminology: Premature Late premature Newborn Neonate Infant

Premature: <34 weeks gestation Late premature: 34-36 6/7 weeks gestation Newborn: Birth to 48-72 hours Neonate: 28 days Infant: 1-12 months

HSV diagnostic studies

Presence of painful grouped vesicles on erythematous base usually allows clinical diagnosis, but can do Viral culture, fluorescent antibody staining or PCR. in infants, any lesion found should be cultured.

Neonatal sepsis risk factors

Preterm Membrane rupture: before labor onset or prolonged>12 hours Chorioamnionitis or maternal peripartum fever Group B strep, UTI or STDs in mother Multiple pregnancy Hypoxia or Apgar score <6 Poverty or age of mother <20

Sebaceous hyperplasia

Prominent yellow-white papules over cheeks, nose and forehead. Prominent yellow-white papules over cheeks, nose and forehead

What does reduced impairment and reduced risk look like?

Reduced impairment: •Prevent chronic symptoms •Require less than 2 x per week use of SABA( not including exercise related bronchospasm) •Maintain near normal PFT •Maintain normal activity levels/attendance •Meet patients expectations of asthma care Reduced Risk: •Prevent recurrent exacerbations/ED visits/Hospitalizations •Prevent loss of lung function/reduced lung growth in children •Provide optimal pharmacotherapy with minimal or no adverse effects of therapy

Health benefits of breast-feeding

Protects baby against disease, provides antibodies, and nutrients. Helps Mom: Decreased postpartum bleeding and more rapid uterine involution, Decreased menstrual blood loss and increased child spacing (lactational amenorrhea), Earlier return to pre-pregnancy weight, Decreased risk of breast and ovarian cancers, Lower risk for developing high blood pressure, cardiovascular disease, and type 2 diabetes

You are overseeing the care of a 24-month-old child who will need to be admitted for 4 weeks of IV antibiotic therapy. How will you promote the healthy development of this child? What interventions will you suggest to the nursing staff? What activities may you want to discourage?

Provide blocks, coloring books, books to read to the child, utensils to feed themselves, soft balls or objects to throw and bounce (safely) Allow them to assist in everyday tasks such as washing hands, feeding, dressing, bathing Consider drowning risks Allow them to make as many choices as possible when appropriate What to wear, choices in food, etc. Discourage excessive media use (greater than 1 hour per day) Encourage high quality programming Consult Certified Child Life Specialist

diphtheria what causes it? incubation period

Rare infection of the resp tract caused by toxigenic strains of gram + C. diphtheria or less commonly Corynebacterium ulcerans. Humans are the only reservoirs and can spread it through droplets or contact with skin lesions (no cases in US since 2003). The incubation period averages 2 to 5 days with range of 1 to 10 days. Incidence greater in the fall and the winter.

Drug eruptions etiology clinical findings and tx

Reaction to medication especially PCN, cephalexin, erythromycin, sulfa, NSAIDs, barbiturates, phenytoin, etc. symmetric macular, erythematous to papular confluent morbilliform rash. intense itching, patches of normal skin throughout. begins on trunk, extends distally, including palms and soles, face with confluent erythema. stop drug, label as allergen to child, give antihistamine and antipruritic. give prednisone if severe, lubricate skin. rash can last 7-14 days.

herpes zoster (shingles): etiology, S/S, diagostic, Treatment

Reactivation of letent varicella infection (post vaccine or active disease) at any time in life. S/S: grouped vesicles in a dermatome distribution, vesicles do not cross midline. itching, pain, occasional low grade fever Tx: antivirals modify its course if started early. immunosuppressed patients may have systemic involvement and require systemic antivirals.

Best way to promote language development

Reading together every day

42-48 months language

Recognizes coins, beings to understand future and past tenses, tells stories, repeats four numbers, gives age, can explain what, who, where, and why

Popsicle Panniculitis

Red bumps on babies cheeks, contender and firm caused by frostbite on the inside of the cheeks. harmless condition that almost always clears up on its own within a few weeks. Children who have had popsicle panniculitis should have their exposure to cold limited until they're a little older and less at risk of a recurrence of the condition.

Meningococcal disease epidemiology and manifestations

Refers to any disease caused by bacterium N. meningitides. Peak ages: children <2 years, adolescents 15-19. increased if living in military barracks or dorm rooms. Most infections in children <5 are due to serogroup B clinical manifestations ranging from transient fever and bacteremia to fulminating disease with death ensuing within hours of onset of clinical symptoms. its the leading cause of meningitis in the US. associated with high mortality and morbidity.

Rashes that have toxic looking accompanying symptoms

Rocky mountain spotted fever, kawasaki disease, steven johnson, meningococcemia, staph/strep scalded skin

For pediatric asthma, before initiating therapy assess ___? The goal of therapy is ___?

SEVERITY: intrinsic intensity of the disease process. assessed to initiate therapy, includes both current impairment and future risk. TO REDUCE IMPAIRMENT AND REDUCE RISK Control: degree to which S/S are minimized by therapy. goals of therapy are met if in control. includes both current impairment and future risk.

24-30 months language

Seems to understand most of what is said, two- to three- word sentences, asks simple questions, identifies objects when given a function (wear on feet)

Perinatal diagnostic testing for mom

Serology: ABO, Rh factor, rubella titres, HIV, HBsAg, syphilis, HSV 1 or 2 serum HSV exposure serology can indicate past exposure or current: It would be important to know if active lesions present, first outbreak, on acyclovir. HBsAg- Hepatitis B surface Antigen - positive or reactive -Infant would need Hepatitis B and HbIG with first 18 Hours of life Gonorrhea, chlamydia (hx and treated/partner) Prenatal testing: Chorionic villi sampling, amniocentesis, fetal ultrasound, genetic testing, GBS screening at 35-37 weeks GA

Acute asthma exacerbation at the hospital

Short acting beta-agonist (continuous albuterol) Systemic corticosteroid: decadron Magnesium sulfate: causes bronchodilation due to smooth muscle relaxation. Continuous infusion terbutaline: risk of tachycardia, hypotension, MI Theophylline: improve response to steroids. Ketamine: potent vasodilator Ipratropium: anticholinergic bronchodilator Epinephrine: SQ or IM, delivers med to small aiways Heliox: lighter than room air, can overcome turbulent airways, improve deliver or meds to obstructed airways due to lower density/ resistance

Pertussis focus on infants specifically (what's different?)

Short or absent catarrhal stage during which the infant can appear deceptively well with a mild cough and no fever, watery coryza, or sneezing. Paroxysmal stage characterized by gagging, gasping, eye bulging, vomiting, cyanosis, and bradycardia (or tachycardia if illness is severe); the cough may or may not be paroxysmal Close contact (usually a family member, often the mother or a sibling) with a prolonged cough and no fever. Complications, including apnea, seizures, respiratory distress, pneumonia and pulmonary hypertension, hypotension/shock, renal failure, and death (infant mortality rate 1%)

Anticipatory guidance for toddlers and preschoolers

Sibling rivalry - make sure you are carving out a little one on one time for your toddler or preschooler when you bring home a new baby - and understand that this sibling rivalry is a normal and expected part of development "picky eating" is expected, give them nutritious options Changes in eating habits stem from developmental changes - decrease in appetite, easily distracted, more curiosity about what is going on around them than what they are eating, desire to use motor skills rather than sit and eat

Positions for breastfeeding: Side-lying position and physiologic laid back position

Side-Lying position: Mother lies on her side, cradles infant in her elbow, and supports the infant's back and neck. A pillow should be arranged under the mother's head and shoulders and a rolled blanket behind the infant's back to keep the infant in side-lying position. This position prevents pressure on abdominal incisions and allows the mother to rest while feeding. Physiologic Laid-back Position: The mother reclines in a position that supports her back and neck and brings her baby to her chest and allows the baby to rest its cheek against her breast. The baby can use the rooting and sucking reflexes to find the nipple, and the mother can support her breast to help guide the baby. When the baby has latched, gravity helps to maintain a deeper latch and the mother is able to relax

48 months gross and fine motor

Skips, catches bouncing ball, stand on 1 foot, walks down stairs (alternating feet), throws ball underhand Strings small beads, demonstrates "handedness", undresses self, buttons, pours from a pitcher

Haemophilus influenzae type B: epidemiology and manifestations.

Small, gram negative facultative anaerobic organisms. Hib is the most virulent of 6 known serotypes. Ages most affected: < 4 years of age. 99% decrease in overall Hib disease in < 5 years of age achieved via vaccination. Clinical: pneumonia, meningitis, epiglottitis, cellulitis, otitis media, septic arthritis, purulent pericarditis.

15 months gross and fine motor

Stands alone, walks, climbs stairs (without alternating feet). puts block in cup, drinks from cup, holds utensils and attempts to use

Erythema toxicum

Starts at 24-48 hours. Firm, yellow- white 1-2mm papules or pustules with surrounding erythematous flare. Clustered in several sites. Incidence as high as 50%, more often in term than preterm infants. No treatment, resolves on own. important to educate parents rash causes no pain, irritation or itchiness. Diagnostics: Wright stain shows large # of eosinophils/cultures are sterile

Stepwise screening program for congenital heart disease and CCHD Pulse oximetry screening algorithm

Step 1: Mid-trimester fetal anatomy fetal ultrasound scanning Step 2: Newborn physical examination Step 3: pulse oximetry CCHD Pulse oximetry screening algorithm Pulse oximetry when infant is 24 hours old or before discharge Measure on right hand and either foot Passes if >95% on both Fails if <90% on either or if >3% difference. If 90-95 on either or if >3% difference, repeat in 1 hour → May require 3 screens All babies who fail the screen should undergo and ECHO by peds cardiology prior to hospital discharge.

Erysipelas

Strep pyogenes cellulitis with pain and elevated advancing border. (picture of baby's cheek) Treat with PCN 1st and Cephalosporin (Ceftriaxone)

Occult bacteremia causes

Streptococcus pneumonia is still most common Peak prevalence between 6 and 24 months• Association with high fever(39.4oC or 103oF)• High WBC count(>15,000). Absence of evident focal soft tissue infection. Neisseria meningitidis, Haemophilus influenzaetype b, E coli and salmonella account for most of the remaining cases. Recent observation is that MRSA is presenting with bacteremia observed before localizing infection

Pneumococcal disease: epidemiology

Streptococcus pneumonia's (pneumococcus) is a leading cause of serious illness among children worldwide. its gram positive, catalase negative. It colonizes that upper respiratory tract. Greatest risk groups: ages <5 and >65. 3 or 4 doses of the vaccine are between 71 and 93% effective at preventing severe disease. increased from 7 to 13 valent in 2013 to cover more invasive serotypes.

Breast feeding failure jaundice: what is it? what causes it?

Suboptimal breastfeeding compared with formula feeding is associated with an increased risk of jaundice and kernicterus. Typically occurs within the first week of life. Associated with the following as predictors of lactation failure in infants with kernicterus: inadequate education from clinicians and lactation consultants, documentation of infant latching, measurement of milk transfer, recording of urine output and stool pattern change.

NP role in breastfeeding

Support moms who want to breastfeed in the hospital and initiate support within 1 hour of birth. Give newborns nothing else to eat or drink besides breast milk. Practice rooming in. Give no artificial teats or pacifiers until breastfeeding is established. Understand how to use breastfeeding equipment and be able to support women who wish to return to work or school while breastfeeding.

A mother brings her 6 week old infant into the clinic for her 1 month old well child physical. the mother exclusively breast feeds and expresses concern that her infant only stools 1 time every 2-3 days. the PNP should?

Tell mom that this is normal, but still do a weight check to make sure the infant is gaining weight.

Young infant fever

The febrile young infant gives few clues to underlying illness. Results of missing signs of infection can be catastrophic Current practice is aggressive laboratory evaluation for the infant under 1 month of age Most patients are hospitalized and given broad spectrum antibiotics which may include acyclovir Consequences: This practice results in substantial iatrogenic morbidity including exposure to hospital acquired infections, and huge stress to families, also expensive

Calcaneovalgus foot

The heel bone is valgus, top of foot is touching tibia, foot looks flat Most common positional foot abnormality, related to intrauterine positioning. 5% of all newborns, most common with decreased space in utero, twins, LGA, first born.

Benefits of fever

The hypothalamus will not allow the temp to rise above 41.5oC(107oF). WBCs work best and kill the most organisms at 38-40oC(100.4-104oF). Neutrophils make more superoxide anion, and there is more and increased activity of interferon. Coxsackie and polio virus replication is directly inhibited.

Identification of septic shock

The inflammatory triad of fever, tachycardia, and vasodilation is common in children with benign infections. Septic shock is suspected if they also have mental status change. The clinical diagnosis of septic shock is made in children who have: a suspected infection with hypothermia or hyperthermia and tachycardia and have clinical signs of decreased perfusion, including decreased mental status, prolonged capillary refill of 2 secs (cold shock) or flash capillary refill (warm shock), diminished (cold shock) or bounding (warm shock) pulses Hypotension is a late finding.

Tinea capitus therapy and adjunctive therapy

Treatment: systemic therapy (topical anti fungal therapy does not penetrate hair shaft). Griseofulvin approved by FDA (25 mg/kg/day for 6 to 8 weeks is current practice by pediatric dermatologists). Terbinafine approved for >3 years of age. Cure rate is going down each year with griseofulvin and is currently at about 75%. Adjunctive therapy: shampoo- either 2% ketoconazole or 1% selenium sulfide reduces shedding of spores to reduce transmission. may also be used on asymptomatic family members. Steroids: use in Kerion not shown in literature to shorten course but still used by dermatologists if patients are not improving after several weeks. Intralesional steroid not recommended. Community cures: Glover's mange, Blue star ointment (camphor and salicylic acid). Shaving head, long term restriction of school attendance.

antipyretics doses and education

Tylenol 10-15mg/kg every 4 hours (rectal tylenol same dose although some recommendations are for higher dose) Motrin 10mg/kg every 6 to 8 hours. not recommended in infants <3-6 months. Not necessary to alternate tylenol and motrin. febrile seizure is unlikely to be prevented by use of antipyretics. tepid sponging may make child less cranky. Treatment of fever is for comfort, to maximize fluid intake and for ease of clinical assessment. Do not create a perception for parents that fever is dangerous and needs to be "controlled" or indeed that is controllable. Normalizing temperature with meds does not make a serious bacterial illness less likely.

Who vs CDC growth charts, when is it appropriate to use each?

Use WHO for kids <24 months Use CDC for kids > 24 months

Who is at risk for contracting pertussis and how long is the incubation period?

Usual source of infection is contact with an adult family member who has a cough without a diagnosis of pertussis Up to 80% of household contacts acquire the disease from an index case due to waning immunization immunity and unrecognized disease Highest risk of morality - infants < 1 month incubation period of 7-10 days (range is 5 to 21). most contagious during the catarrhal stage and first 2 weeks following cough onset

Sepsis treatment

Venous access, blood culture, lactate, pH, and VBG/AGB. Fluid challenge in increments of 20cc/kg with reassessment after each bolus. minimum of 60cc/kg within the first 1 hour. in patients wit cardiac issues, limit boluses to 10cc/kg with careful reassessment. Prepare pressor if not responding after second bolus. oxygen appropriate IV antibiotics, and consider steroids if patient has adrenal suppression. Consider early intubation (reduces WOB). continuous presence of nurse and doctor at bedside frequent reassessment.

Hepatitis B epidemiology: transmission + maternal infant transmission info

Vertical from mother to child, during transfusions, sexual transmission (common). Percutaneous inoculation per shared needles or other drug-preparation among IV drug abusers, contact with blood or open sores of an infected person, needle sticks or sharp instrument exposures, sharing items such as razors or toothbrushes with an infected person. Maternal infant transmission: The infection rate of infants born to hepatitis B surface antigen (HBsAg)-positive mothers is as high as 90 percent among infants who do not receive hepatitis B immune globulin and hepatitis B vaccination at birth. Mother-to-child transmission may occur in utero, at the time of birth, or after birth. However, most infections occur at the time of birth.

CBC (complete blood count)

WBC (norm: 5,000-15,000) elevated in bacterial, decreased in viral. Diff: neutrophils/bands increased in bacterial. Chronic inflammation may cause anemia. Thrombocytosis common during acute infection. Lymphocytes dominate for young children because lymphocytes fight viruses the most. And kids are exposed to viruses. Increase in neutrophils if bacterial and increase in lymphocytes if viral. (Try to determine if its viral or bacterial based on what type of WBCs) HGB/HCT: not usually affected by acute infectious disease, commonly low in chronic inflammatory processes. PLT: may increase with inflammation or decrease if viral suppression of bone marrow.

Markers for SBI in young infant (less than 3 months)

WBC: <5000 or >20,000, PMN <4000. Band count >1000. elevated procalcitonin levels is a predictor of SBI increasingly available. elevated liver enzymes suggests HSV

the best ways to manage mastitis include?

Warm compress, fully empty breast, rest, and increase fluids.

poor weight gain (breastfeeding complications). warning signs and contributing factors

Warning signs: Continued weight loss after 5-7 days old. Failure to regain birthweight by 2-3 weeks old. Failure to maintain ongoing weight gain of 0.5 to 1 oz per day. Weight below 3rd percentile for age. Lethargic, sleeping, inactive, unresponsive infant, dry mucous membranes & poor skin tugor. Newborn sleeping longer than 4 hours between feedings Contributing factors: Infrequent or inadequate feedings (Short time at the breast, ineffective latch, not following hunger cues, long stretches of sleep, fewer than 8 feedings in 24 hours). Inadequate milk production. Genetic predisposition, infection, or organic disease. Physical anomaly that may interfere with suckling or swallowing. Maternal education deficit

imaging studies: xray, CT, MRI

XRAY:useful for specific issues, least expensive, readily available CT: best for detecting calcification and fresh blood, limit due to radiation exposure. MRI: detects neuronal migrations, soft-tissue lesions, brain abnormalities, ventricular size, excellent soft tissue images with great detail, often requires sedation, advanced imaging available, expensive

Caput succedaneum:

a diffuse, superficial swelling of the soft tissue of the scalp with possible underlying bruising. The swelling crosses the suture lines, but it can also be very localized. Edges poorly defined, subcutaneous fluid beneath scalp. (Often associated with primigravida and traumatic delivery. No treatment necessary, swelling resolves within a few days. Monitor for jaundice as blood is reabsorbed)

Human papillomavirus: epidemiology

a sexually transmitted pathogen that causes anogenital and oropharyngeal disease in males and females. most common STI. There is evidence linking HPV to cervical carcinoma is extensive. virtually all cases of cervical cancer are attributable to HPV, it also causes genital warts, anal cancer, and penile cancer.

What is the most common neurobehavioral disorder of childhood? a. Attention deficit hyperactivity disorder (ADHD) b. Depression c. Oppositional defiant disorder (ODD) d. Anxiety

a. Attention deficit hyperactivity disorder (ADHD)

early sepsis or SIRS (systemic inflammatory response syndrome) is defined by presence of:

abnormal temp >38.5 or <36 or abnormal WBC count. AND abnormal HR or abnormal RR for age.

bronchiolitis clinical manifestations

accompanying symptoms: mild conjunctivitis (30% of cases). pharyngitis, otitis media, mucosal inflammation (congestion, rhinorrhea, sneezing), lower resp tract involvement (cough, increased WOB, subcostal/intercostal retractions, expiratory wheezing, coarse crackles, abdominal distention, hepatospenomegaly). we see accessory muscle use because the child is trying to overcome the resistance of obstructed airways Auscultation: Vibration of conducting airways, prolonged expiratory phase, diffuse polyphonic wheezing (4-5 days into illness), coarse crackles scattered throughout bilateral lungs (intrathoracic problem) Hypoxia: Ventilation-perfusion mismatch secondary to mucous plugging Carbon dioxide retention leading to Respiratory acidosis infant specific: lethargy, poor feeding, fever (low grade to moderate <38.9), otitis media, apnea (20%). course is at it's worse by 48-72 hours after wheezing onset and after should improve.

C-reactive protein (CRP)

acute phase reactant, serious bacterial infections lead to increased CRP. May also be elevated due to trauma, rheum diseases, oncologic dx

admission criteria

admit any infant <30 days. consider strongly for infants 30-60 days. Toxic looking infants are worked up and admitted. Admit with IV antibiotics Spinal tap when indicated, LP should be performed in and infant <30 days and in children give IV antibiotics for fever without focus.

What type of isolation is needed for patients with pertussis?

aerosol droplet transmission.

chlamydia trachomatis pneumonia: onset/age, clinical findings, diagnostic, treatment

age: 2-19 weeks (1-3 months). onset is gradual. clinical findings: tachypnea, staccato cough, crackles, wheeze, conjunctivitis (50%). afebrile. diagnostic: organism is c. trachomatis, hyperinflation and infiltrates on xray. treatment: erythromycin

Meningococcemia

aka sepsis, may occur with or without meningitis. symptoms: fever, petechial or purpuritis rash, hypotension, shock, acute adrenal hemorrhage, multi-organ failure

Roseola infantum etiology, findings, treatment

aka: 6th disease, exanthem subitum Etiology: Human Herpes virus 6, type A & B Clinical findings: 1. Incubation period 12 days 2. Prodrome of high fever (103-106F), otherwise relatively asymptomatic 3. Rash develops after fever breaks. diffuse, nonpruritic, discrete, rose colored maculopapular. fades with pressure. Treatment: treat fever, as this is common cause of febrile seizures

Herpes Simplex Virus 1 vs 2

among the most widely disseminated infections in humans. HSV1: usually associated with infection of mouth, lips, eyes, and CNS. HSV2: usually associated with genital and neonatal infections (75% of neonatal infections)

All the following are appropriate screening tools for a child who is 9 years old EXCEPT: a. Pediatric Symptom Checklist (PSC) b. Patient Health Questionnaire (PHQ-2, PHQ-9, PHQ-9 Adolescent) c. Screen for Child Anxiety-Related Disorder d. NICHQ Vanderbilt Assessment Scale

b. Patient Health Questionnaire (PHQ-2, PHQ-9, PHQ-9 Adolescent)

Impetigo S/S, Treatment

bacterial skin infection characterized by isolated pustules that become crusted and rupture topical antibiotics (mupirocin or bacitracin) if localized and systemic antibiotics (cephalexin, augmentin, clinda) if more widespread.

Neonatal Abstinence Syndrome (NAS): what is it? how to score? how to manage?

an array of signs and neurological, GI, autonomic behaviors in newborn after abrupt discontinuation of Opioid drugs taken by the mother. Types of drugs: Gestational opioid exposure (including methadone): symptoms within the first 72 hours of birth., Buprenorphine: decreased NAS when compared to methadone, symptoms occur later than with methadone. A partial mu-opioid agonist., Benzodiazepine: symptoms can occur a few days to 3 weeks after birth and may last several months Finnegan scoring system: developed to assess opioid-induced withdrawal. CNS dimensions, metabolic/vasomotor, GI Treatment: opioid containing med (morphine sulfate, methadone), some newer meds under evaluation (buprenorphine, clonidine). Non-pharmacologic: soothing, swaddling, non-nutritive sucking Current standard of care: transfer infant to a specialized NICU, assess using finnegan neonatal abstinence scoring system (FNASS), and treat according to the score. Round the clock dosing with scheduled tapering, typically over 30 days. Emerging model of care: keep infant with mother, assess infant ability to eat, sleep, and be consoled (ESC scale), coach mother on how to soothe infant and breastfeed. Use morphine as needed instead of on a scale.

Plagiocephaly

an asymmetrical, flattened deformity of the skull. Synostotic: rare, due to premature fusion of one or more of the sutures of the skull. Nonsynostotic: positional or deformational plagiocephaly (DP) because it results from external forces that mold the skull in the first year of life. Increased incidence since the Back to Sleep campaign (1972). Occurs most often as: lateral DP→ (flattening of the side of the occiput) or Central DP/ brachycephaly (shows distinct occipital flattening with increased head width or height, back of head) DP affects 1:5 infants in the first 2 months of life. More severe if torticollis or gross motor delay. If fixed sutures, suspect craniosynostosis. The peak prevalence of DP is 4 months of age. Management: helmet, 23 hours per day, 2-6 months of age.

UTIs: who is at risk? how to rule out?UA

anybody less than 6 months with fever, females less than 2 years without a source. Do not rule out UTI based on a negative UA, obtain a culture if you are going to do a UA in a child <2 years to eval fever. urine culture: catheterized or suprapubic aspiration

what is the most common chronic pulmonary disorder in children?

asthma Incidence: 1 in 12 kids between 0-17 years old diagnosed with asthma in 2016 Symptoms may interfere with normal pediatric activities such as sports, school trips, gym class, play, brass/wind instruments Asthma is the leading cause of school absences, students may miss >/= 10 days of school per year (severe asthma may miss greater than 30 days/year, respiratory symptoms are responsible for 60% of school absences Costs of uncontrolled asthma: increased medications required when not well controlled, lost time at work and regular activities for caregivers, decreased productivity at work/school due to disturbed nighttime activities

febrile seizures

atypical: longer than 15 mins, focal, repeated seizures for several hours or days. treatment: LP indicated if on abx, mental status changes, toxic, if child does not return to preseizure baseline. antipyretics or limited efficacy and reassurance. anticonvulsant therapy and EEG not indicated for simple febrile seizure. Prophylactic oral diazepam or diastat for recurrent febrile seizures on individual basis.

anterior fontanel

average closure 14 months is the largest and most important for clinical evaluation. The average size of the anterior fontanel is 2.1 cm, and the median time of closure is 13.8 months. Delayed closure/ larger fontanel: achondroplasia, hypothyroidism, Down syndrome, increased intracranial pressure, and rickets. Premature closure: craniosynostosis, microcephaly, FAS, intrauterine infections, chromosomal abnormalities, Hypoxic-ischemic encephalopathy, or could be a normal genetic variation

newborn hypoglycemia: what is it? symptoms? what makes it worse? how to assess manage it?

blood glucose levels as low as 25-35 mg/dL common in the first 1-4 hr for healthy neonates, usually asymptomatic and transient. Generally above 35-45 mg/dL by 4-24 hrs of age. Symptoms: Jitteriness/tremors (most frequent symptoms), hypothermia, hypotonia, apnea or cyanotic spells, grunting and tachypnea, irritability, lethargy or stupor, sweating, tachycardia. Maternal medications that effect: terbutaline sulfate or ritadrine (preterm labor), beta blockers or beta adrenergic agents (HTN, arrhythmias), Benzothiazide diuretics (HTN), oral hypoglycemic agents (maternal diabetes). IDM (infants of maternal diabetes): frequent cause of hypoglycemia for infant, greater chance for BS instability for infant, 50% will also have hypocalcemia and hypomagnesemia within 48-72 hours of birth. Screening: Who? Small for gestational age, mother with diabetes, late pre-term infants, large for gestational age, maternal/ fetal conditions that place the infant at risk, clinical symptoms in term infant. Breastfed infants may tolerate lower serum glucose better than formula fed How often to check: Check BS1 hour old , 2 hours, 4 H, 6H, 12 H and 24 H of age. Birth to 4H acceptable BS 25- 35 mg/dL 4 H to 24 H acceptable BS 35- 45 mg/dL Management:Glucose gel x 3 for first 24 h. breast feed (q 1.5-2 hours). bottle feed (q 3 hours) with increased volume. If used up, go to NICU for IV glucose. Infants eligible for DC home last BS above 60. Or last two BS above 50.

Monitoring physical development and growth of toddlers and preschoolers

blood pressure monitoring at age 3. Recumbent length until 24 months. Height after 24 months Head circumference until 24 months

All of the following are true regarding breastmilk jaundice except?

breastfeeding should be discontinued

Clinical findings suggesting a newborn GI obstruction and what diagnostic tests do you order?

by 24 hours old, expect 1 BM (mec at delivery does not count). Pay attention to consistency of emesis/stool. Bilious emesis in a newborn can occur without intestinal obstruction. Attempt to pass the NG tube to confirm esophageal patency. Get an abdominal AP radiograph, it would show: Duodenal atresia (double bubble) Meconium ileus (distended bowel loops with ground glass appearance) NEC (dilated bowel loops, pneumatosis) Intestinal obstruction (dilated bowel loops proximal to obstruction and paucity of gas distal to obstruction)

A healthy 11-year-old male presents to your clinic in need of immunizations. His past immunizations are up to date. What vaccinations do you anticipate giving this patient? a. Tdap, influenza, MMR, varicella b. HPV, polio (IPV), Tdap, influenza c. Tdap, influenza, HPV, meningococcal conjugate d. Tdap, influenza, HPV, serogroup B meningococcal vaccine

c. Tdap, influenza, HPV, meningococcal conjugate

Infants 1 to 3 months who are nontoxic and febrile. what do they need?

can be safely treated as outpatient. 2 options: 1. blood, urine cultures, LP. give cefriaxone IM and return for level within 24 hours. 2. blood and urine cultures and careful observation

Managing acute exacerbations at home

categorize as mild, moderate, or severe based on symptoms and PEF Initial therapy: ◦Inhaled SABA (up to two treatments 20 mins apart) ◦May consider every 3-4 hours if effective for 24-48 hours ◦If symptoms persist, seek additional help and initiate oral systemic corticosteroid ◦If poor response, progressive symptoms, repeat SABA, add corticosteroid, proceed to ED/911

infectious mononucleosis: causative agent, manifestations, diagnostic, mangement

causative agent: epstein barr (family of herpesvirus). incubation prior to developing symptoms is 4-8 weeks. Clinical findings: fever, tonsillar pharyngitis, and lymphadenopathy. disease of primary lymph nodes and peripheral blood. malaise, headache. hepatomegaly, splenomegaly, skin rash. diagnostic: lymphocytosis >4500. (more than 10% of total lymph nodes). confirm with EBV antibody test (mono spot). Manage: supportive with adequate bed rest, corticosteroids or acyclovir not recommended in non-complicated cased. PCN/amoxicillin contraindicated (can cause rash). contact sports restricted until splenomegaly resolves.

Generalized head to toe rashes

chicken pox, measles, scarlet fever(infections, immune reactions). idiopathic thrombocytopenic purpura.

seborrhea

chronic inflammation often called "cradle cap" in infants and "dandruff" in adolescents. related to overproduction of sebum. findings: for infants, erythematous, flaky to thick crusts of yellow, greasy (waxy appearance) scales predominantly on the scalp, but also on the face, behind ears, on neck, and trunk/diaper area. in adolescents, mild flakes with some red and yellow greasy scales on the scalp, forehead, and nasal bridge and eyebrows. not itchy, no pustules. Treatment: antifungals (azoles, selenium sulfide), anti-inflammatory (topical steroids, topical calcineurin inhibitors). medicated shampoos (ketoralac) *Infants is usually self limiting

36 months gross and fine motor

climbs ladder, tricycle, balance on 1 foot, catches a ball Builds 9-10cube tower, imitates 3 cube bridge, copies circles, uses scissors, brushes teeth, puts on shoes, feeds self, twists a jar lids

Bronchiolitis diagnostic evaluation

clinical diagnosis based on history and PE. no X-rays are required for diagnosis. may confirm with PCR, but not truly necessary to perform any testing. assess for risk factors of severe disease: age <12 weeks, hx of prematurity, underlying cardiopulmonary disease, immunodeficiency

Pneumococcal disease manifestations

community acquired pneumonia, empyema, AOM, sinusitis, sepsis, bacteremia, meningitis Bacterial pneumonococcal pneumonia: pleuritis chest pain, cough productive of mucopurulent drainage, rusty sputum, dyspnea, tachypnea or tachycardia, hypoxia, malaise or weakness, fever. ***most common bacterial infection <5: prescribe high dose amoxicillin (most common bacterial infection over age 5: mycoplasma pneumonia.)

Physical developmental changes for toddlers and preschoolers

complete set of teeth by age 3. Visual acuity 20/30 (begin visual acuity testing at 3 years old). Hearing maturity by age 4. the diaphragm matures which causes abdominal respiration moment to decrease

Step 4: severe persistent asthma symptoms and lung function

continual symptoms, SABA several times/day. extremely limited physical activity, frequent exacerbations nighttime symptoms: often seven times/week FEV <60% predicted

Acute hemorrhagic conjunctivitis

coxsackie A. sudden eye pain, photophobia, blurred vision, tearing, conjunctival erythema, most patients recover within 2 weeks.

All of the following are included in the diagnosis criteria for attention deficit hyperactivity disorder (ADHD) EXCEPT: a. DSM-5 criteria met b. Rule out alternative causes c. Documentation of symptoms/impairment in more than one major setting with information obtained primarily from reports from those involved in the child's care d. All of the above are included in the diagnosis criteria for ADHD

d. All of the above are included in the diagnosis criteria for ADHD

Maternal breastfeeding diet

daily caloric intake of 300 extra calories. Avoid caffeine in excess Refer to WIC: they offer food to breastfeeding mother's because she does not need formula

consider herpes and add acyclovir especially if...

delivered vaginally, premature rupture of membranes, fetal scalp electrodes, skin/eye/mouth lesions, seizures or ill looking, CSF pleocytosis

Eczema herpeticum: etiology, S/S, diagostic, Treatment

disseminated herpes virus infection at sites of epidermal disruption (eczema, burns, atopic dermatitis). potential for life threatening systemic infection. Tx: inpatient IV antivirals (acyclovir)

Fever without infection:

does not directly increase the body's set point. CNS disease that directly affects the hypothalamus is intracranial hemorrhage and infection. Diseases that increase the body's production of heat: hyperthyroidism, malignant hyperthermia, salicylate overdose. Excess heat load: infant left in a closed car, over bundled, or left next o a heater for too long. Defective heat loss mechanisms: burns, heat stroke, drugs that compromise blood flow and sweating mechanisms.

What to ask for a chief complaint of rash?

duration, distribution, associated symptoms, meds/creams/treatments that have been tried, whether any other family member has a similar rash or related symptoms. If the family member with a similar rash is with the child, check the family member as well.

All of the following are appropriate anticipatory guidance for the PNP to offer caregivers of school age children EXCEPT: a. Discipline children by removing books or writing supplies b. Monitor all digital and social media use and turn off screens 15 minutes before bedtime c. Encourage participation in meal planning, food shopping/selection, and meal preparation d. Provide opportunities to ask questions about sexuality, drugs, alcohol, and tobacco e. Both A and B

e. Both A and B

Neonatal infection causes for early onset (<7days) and later onset (>7days)

early onset: e. coli, GBS, listeria monocyogenes, enterobacteriaceae, enterococcus, strep viridian, strep pneumonia, coagulase neg. staph, herpes simplex. Late onset: e. coli, GBS (decreasing), listeria monocytogenes, enterobacteriaceae, staph aureus, strep pneumonia, neisseria meningitides, hemophilus influenza, herpes simplex.

Missed diagnoses for scalp symptoms

eczema (b/c scalp is unusual as an isolated finding) Folliculitis/traction alopecia: most difficult entity to distinguish from in differential diagnosis. pattern of hair loss may be helpful (tinea is more patchy) may need a culture to distinguish. seborrhea or dandruff: rarely occurs in post neonatal and prepubescent kids. Abscess: no associated scale, patient may be sick or febrile, avoid I&D of ringworm lesions as this procedure aggravates and extends infection. Psoriasis

Patchy or uniform

eczema is patchy, scarlet fever is uniform

Occult bacteremia treatment

empiric antibiotics should be targeted against s. pneumonia, n. meningitides, and h influenza, consider e coli. Treat with: amoxicillin, augmentin, bactrim 2nd or 3rd gen cephalosporin, or single dose ceftriaxone 50-75mg/kg FOLLOW up!

pityriasis rosea: etiology, rash, treatment

etiology is unknown. oval shaped lesions with scale around lesions (herald patch). Can have a christmas tree presentation: rash especially on back, follows dermatome lines with oval lesions running parallel and wrapping around the trunk horizontally. Frequently pruritic, predominantly on the trunk, extensive rash may include face and extremities. no specific treatment: calamine lotion, tepid baths, antihistamines, aveeno lotion to ease symptoms. could prescribe topical corticosteroid for itching. oral erythromycin may hasten the resolution of eruptions.

What is a sign of poor weight gain in a breastfed infant?

failure to maintain ongoing weight of 0.5-1oz per day

History to obtain for a child's suspected or known asthma

family, conditions associated with asthma, complaints of chest tightness/dyspnea, coughing/wheezing with what activities, pattern of symptoms, recurrent respiratory diagnoses, aggravating factors, level of alertness.

Hepatitis A Epidemiology/ transmission, manifestations

fecal-oral transmission or consumption of contaminated food. Viral replication in the liver. virus present in the blood and feces 10-12 days after infection. virus excretion may continue for up to 3 weeks after onset of symptoms. Incubation period 15-50 days. Course is indistinguishable from that of other types of acute viral hepatitis. typical abrupt onset of fever, malaise, anorexia, nausea, abdominal discomfort, dark urine, and jaundice. usually resolves within 2 months. 10-15% of persons have prolonged or relapsing signs and symptoms for up to 6 months.

Infant sepsis syndrome: what 3 things?

fever >39, ANC >10,000, age 3-36months. If a child meets all 3 criteria, he has a 2% risk for sepsis. if untreated 3% will progress to meningitis. Bacteremia risk peaks at 8-12 months.

Fever vs. hyperpyrexia

fever is bodies physiologic defense mechanism and is 38C or higher core temp. Hyperpyrexia is >41 degrees Rectal temp is gold standard

aseptic meningitis

fever, headahe, stiff neck, seizures, altered sensorium. PCR recommended. CSF would show normal glucose, no bacteria, elevated protein, and lymphocytosis. Enteroviruses (coxsackie, echovirus) are responsibel for > 90% of cases.

Follow-up and return for...

follow up within 24 hours for children on antibiotic and 48 hours for others. return for positive cultures: + pneumococcal bacteremia, repeat culture and oral antibiotics, if still febrile, full sepsis work up. neisseria, h flu, gram negative rods staph aureaus need IV antibiotics and cultures. Well appearing UTI, treat with oral antibiotics.

Steven's johnson Sydrome: etiology, clinical findings, tx

frequently drug related (antibiotics and anticonvulsants). but can be caused by viral illness inflammation and peeling of mucous membranes. target lesions with a blister or sore surrounded by a flat red area of skin. eye involvement may lead to visual impairment. secondary bacterial infection common. SJS is < 30% of body surface area. it is considered toxic epidermal necrolysis (TEN) if it covers >30% BSA. Hospitalize, treat like a burn with fluid management and skin care.

What do you do in fever/ presumed sepsis of a newborn <7days?

full sepsis workup including LP and IV antibiotics- ampicillin and gentamicin (or cefotaxime) and acyclovir. IV hydration.

How to diagnostically evaluate for pertussis?

gold standard: culture for B. pertussis (caution: 12-60% sensitive, 100% specific, difficult to obtain, may be negative if ill >2 weeks, previously vaccinated, or treated on abx). PCR: 70-99% sensitive, 86-100% specific serologic tests: can be performed up to 12 weeks after onset of symptoms CBC: leukocytosis with marked lymphocytosis. (in infants and young children, to adolescents) xray: normal or non-specific finding

Pertussis: etiology

gram negative bacillus, bordetella pertussis

treatment for persistent asthma: step 6

high dose ICS + either a LABA or montelukast oral systemic corticosteroids consult with specialist

Low or regular dose amoxicillin vs high dose

high dose" versus "low dose" depends on the minimal inhibitory concentration (MIC) that will inhibit the visible growth Goal: antibiotic concentration that is 4X MIC. GABHS MIC <0.1- 1.0 mcg/mL (penicillin sensitive) To effect cure concentration must reach and exceed the MIC for that organism. This will exceed MICs of all intermediate and most high level resistant S. pneumoniae S. pneumoniae: significant percent have high level resistance (2-4 mcg/ml), so For s. pneumonia: use high dose regimen 90mg/kg/d in 3 divided doses.

Rotovirus: epidemiology and manifestations

highest incidence among children 3 to 35 months of age. infection leads to isotonic diarrhea, first infection usually does not lead to permanent immunity, reinfection can occur at any time. subsequent infections are generally less sever. the vaccine is contraindicated if the baby has a hx of intussusception. Manifests: diarrhea, dehydration, electrolyte imbalance, metabolic acidosis, immunodeficient children may have more severe or persistent disease.

rashes that itch

hives/urticaria, eczema, chicken pox, scarlet fever, contact dermatitis, drug reactions, insect bites (scabies, bed bugs, mosquito bites, bee stings), fungal infections.

Erythema multiforme: etiology, clinical findings, tx

immune meditated hypersensitivity reaction often to infection, especially HSV and mycoplasma Key finding: target or iris lesions (looks slightly purple in the middle of the iris). lesions are fixed, symmetric. typically distributed on hands, feet, elbows, knees, also face, neck, and trunk. (they usually have a history of infection. Identify, treat, and discontinue trigger, treat infection if present, supportive hydration, pain relief, oral antihistamines, cool compresses, prevent secondary infection. lesions last 5-7 days, recur in patches over 2-4 weeks, and should resolve without scarring or sequelae.

Parainfluenza: what does it cause? clinical findings? diagnostic?

important cause of croup, bronchitis, bronchiolitis and pneumonia. Clinical findings: 80% affect upper airway, accounts for 50% of croup (laryngotracheobronchitis), 15% of bronchiolitis and pneumonia. Diagnosis: routine testing not indicated, PCR or fluorescent antibody testing.

Post immunization fever

in infant with fever less than 24 hours after immunization, should consider obtaining urine. infant with fever more than 24 hours, evaluate without consideration for recent immunization status. SBI occur at the same rate in febrile infants more than 24 hours post-immunization as in other infants.

Problems with tinea capitus therapy

increasing resistance to griseofulvin requiring prolonged therapy. Drugs not readily available to any HMO clients. ex) Diflucan, griseofulvin liquid, alternative anti-fungals restricted or rationed.

Classes of asthma pharmacotherapy

inhaled corticosteroid systemic corticosteroid beta agonists: nebulizer, metered dose inhaler +spacer, dry power inhaler *SABA MDI - no need to synchronize breathing with inhalation to activate MDI. Younger kids need mask + spacer. SABA nebulizer - may be preferred during severe obstruction SABA DPI - no spacer or shaking required, not for kids < 4 yrs

classifications of pediatric asthma

intermittent, mild persistent, moderate persistent, and severe persistent. classification determined by: symptoms, recurrences, need for meds, and pulmonary function measurements. Exacerbations may be mild, moderate, or severe in any classification of asthma. Well controlled asthma has an exacerbation once every 3 years.

candida intertrigo what is it? how to treat it?

irruption of the fold of skin in warm, moist body areas; use drying agents like talc, cornstarch, topical antifungals (lotrimin cream), oral antifungals

30 months gross and fine motor

jumps (both feet), climb stairs (alternating feet), stand on 1 foot, walks on tiptoes. builds 9 cube tower, draws vertical/horizontal line, imitates circles, buttons large buttons, holds fork with fist, dresses with assistance.

Bacille Calmette-Guerin (BCG) vaccine

live strain vaccine to prevent TB. Used in countries with high prevalence of TB. BCG can produce a severe hypersensitivity reaction to the mantoux skin test giving a false positive.

Pityriasis (tinea) versicolor signs and symptoms and treatment

located: trunk, neck, and arms Color: pink, coppery brown or pale Mildly pruritic or not at all Common in hot, humid areas and in these who sweat a lot (active teen boys in the summer) Etiology: yeast (malassezia furfur). Treatment: topical antifungals, keratolytics or oral antifungals (ketoconazole). recurrence common.

Treatment for persistent asthma: step 2

low dose ICS alternative: cromolyn or montelukast

Enterovirus rash

macular, maculopapular, urticarial, vesicular, petechial, may mimic rash of meningitis, measles or rubella

appearance and texture vocab: macular papular plaque wheal vesicles bullae pustules sandpaper scaly wet dry

macular: not palpable papular: small raised lesions plaque: larger raised areas wheal: or hives, localized skin edema vesicles: small <.5cm, clear fluid filled lesions bullae: large, fluid filled lesions pustules: small pus filled lesions sandpaper: numerous small papule scaly wet: weeping eczema, intertrigo dry: psoriasis

erythrocyte sedimentation rate (ESR)

measure of inflammation. low rate (<10mm/hr) then bacterial infection is unlikely. if rate is <20-30mm/hr, that supports a viral infection. >50mm/hr consistent with significant pathologic process such as (bartonella/ mycobacteria/ abcesses). ESR lags behind CRP

Spirometry what is it?

measures volume of air an individual inhales or exhales as a function of time. Recommended every 1-2 years, but not performed in >59% of peds practices. important to consider the childs ability to perform the test forced expiratory vital capacity: volume exhaled after maximal inhalation (fast and hard exhalation) normal values depend on heigh, age, gender, and ethnicity.

Treatment for persistent asthma: step 3

medium dose ICS

Treatment for persistent asthma: step 4

medium dose ICS + either a LABA or montelukast consult with specialist

Rubella

mild, maculopapular rash along with lymphadenopathy, and a slight fever. rash starts on face, becomes generalized within 24 hours, and last a median of 3 days. was eliminated from the US in 2004, but common in other areas of the world. If mother is infected during 1st trimester, effects on fetus include: cataracts, heart defects, hearing impairment, and fetal demise.

Assessment of mild, moderate, severe, and impending respiratory distress

mild: end expiratory wheeze, no wheeze. none to minimal retractions. slightly prolonged expiratory phase. normal aeration. able to talk in sentences. Moderate: wheezing throughout expiration, intercostal retractions, prolonged expiratory phase, decreased breath sounds at bases severe: accessory muscle use, inspiratory/expiratory wheeze or no wheeze. suprasternal retractions, abdominal breathing, decreased breath sounds in bases. Impending respiratory arrest: diminished breath sounds, unable to maintain respirations, severely prolonged expiratory phase, drowsy/confused

Acute bacterial pharyngitis and tonsillitis from GAS: findings and scarlet fever

most common bacterial cause in children and adolescents.Accounts for 15-30% of infections in children with sore throat and fever Sudden onset of sore throat, fever >38, malaise, nausea, headache. Throat is red, swollen, with or without exudate, cervical lymph nodes are tender. Scarlet fever: sandpaper rash, blanches with pressure, may become petechial, most intense in groin and axilla, fades in 2-5 days, fine desquamation Face flushed with circumoral pallor, Strawberry tongue Lack of cough or URI symptoms

Meningococcal meningitis

most common presentation of invasive meningococcal disease. incubation 3-4 days. abrupt onset of fever, meningeal symptoms, hypotension, rash, headache, stiff neck.

Enterovirus (non-polio): epidemiology and clinical types

most disease caused by 10-15 serotypes of echovirus and coxsackie. Concentrated in the GI tract as primary site of invasion. Transmission via GI tract, respiratory tract, and vertically (prenatally or postnatally) clinical entities: rash, herpangina, acute lymphonodular pharyngitis, hfmd, aseptic meningitis, acute hemorrhagic conjunctivitis, orchitis, myocarditis or pericarditis (25-35% of cases)

bronchiolitis therapeutic interventions

nebulized hypertonic saline in hospitalized patients only as it may improv mucociliary clearance, oxygen if <90%, antibiotics only if secondary bacterial infections, NG or IV fluids as needed for dehydration indications for hospitalization: infants <2 months old, increasing resp distress such as progressive stridor at rest, apnea, tachypnea (>50-60), hypoxia, restlessness, fallow, cyanosis, increased CO2, inability to tolerate PO, decreased LOC, chronic CV disease, immunodeficiency

HSV management

neonatal herpes: life threatening due to disseminated disease. HSV1: treat with oral valacyclovir or famciclovir may shorten duration of disease for primary or recurrent infection. Infants, persons with eczema, or HIV are at high risk for disseminated disease and should receive IV acyclovir.

bacterial pneumonia treatment: inpatient

neonate: ampicillin + cefotaxime, ceftriaxone, or gentamicin fully immunized infant or school-age child with community-acquired case: ampicillin or penicillin G unless high incidence of S. pneumoniae if high incidence of S. pneumoniae: Third generation cephalosporin like Ceftriaxone or Cefotaxime Add macrolide to beta lactic if M. pneumonia or c. pneumonia present. (azythromycin) Consider vancomycin or clindamycin is s. aureus strongly considered.

AAP recommendations to prevent drowning

never leaving children by themselves, and never leaving them in the care of an older child around pools, bath tubs, spas, wading pools, irrigation ditches, ponds, or other standing water Examples of risks in the home: infant bath seats, buckets, toilets (being left alone in the bathroom), Prevent unsupervised access to bathroom, pool, open water Supervising adult should have swim skills, be able to recognize child in distress, safely perform rescue, initiate CPR, call for help "touch supervision" Supervising adult should not be texting, socializing, consuming alcohol, *there should be a clear hand off ofresponsibility from one watcher to the next 4-foot, 4 sided fence around pool. Gate should be self closing, self latching. Should separate the pool from the house and from the rest of the yard. The majority of drownings occur during a time when the parent did not expect the child to be around the water, thats why so many of the interventions are based on preventing unsupervised access to water parents/pool owners should have CPR skills. Ensure safety of home/business with pool that child will visit US Coast Guard approved life jacket for child & adult in watercraft Know the depth of the water before entering Select swim areas with lifeguards in open bodies of water Recognize drowning risks in cold seasons

UA meaning and indications

nitrite test has higher specificity, LE is more sensitive. clear looking urine on inspection has a 97% negative predictive value. UA and micro together has a sensitivity of 82% and specificity of 92%. presence of bacteria on unseen specimen (93%). concern that unnecessary catheterizations being done in infant fever. following algorithm is helpful.

spirometry obstructive vs restrictive

obstructive: limited expiratory airflow, airways cannot empty as readily as normal airways. ex. asthma, emphysema, CF (FVC normal or decreased, FEV1 and FEV1/FVC decreased) Restrictive: reduced lung volume/decreased lung compliance. ex. interstitial fibrosis, scoliosis, obesity, neuromuscular disease, CF. (FVC decreased, FEV1 decreased and FEV1/FVC normal or increased)

Diagnostic test for tinea capitus?

obtain a culture, to confirm diagnosis, identify any more resistant organisms (ex. microsporum canis requires prolonged therapy). the best opportunity for a good culture is at initial presentation.

Spirometry pre and post bronchodilator

obtain a flow-volume loop, administer a bronchodilator. Obtain the flow-volume loop again a minimum of 15 minutes after administration of the bronchodilator Calculate percent change (FEV1 most commonly used---so % change FEV 1= [(FEV1 Post-FEV1 Pre)/FEV1 Pre] X 100) Reversibility is with 12% or greater change

A fever that accompanies a rash usually?

occurs as a pattern in relation to rash onset. ex. viral exanthems such as roseola, measles, scarlet fever, serum sickness, Kawasaki disease, toxic shock

viral pneumonia: onset, clinical findings, diagnostic, treatment

onset acute or gradual clinical findings: age dependent, cough, coryza, hoarseness, crackles, wheezing, stridor. fever is present but less prominent than in bacterial. diagnostic: CBC normal or slightly elevated WBC, organism RSV, parainfluenza, influenza (on PCR). transient lobar infiltrates on xray. Supportive care

Bacterial pneumonia: onset, clinical findings, diagnostic

onset acute or gradual. (intrathoracic process) clinical findings: age dependent, URI, cough, dyspnea, tachypnea, rales, diminished breath sound, grunting, retractions, toxic, severe distress. Fever >39 diagnostic: CBC elevated WBC >15,000, lobar consolidation on X-ray, if you can get a sputum culture organism is 90% likely streptococcus pneumonia

mycoplasma/chlamydophilia pneumonia (atypical pneumonia): onset, clinical findings, diagnostic, treatment

onset is slow, occurs in kids >5 years old findings include persistent cough, malaise, and headache, fever is acute onset >39. diagnostic: CBC normal, organism is mycoplasma or chlamoydophila pneumonia. variable, interstitial infiltrates on xray. treatment: erythromycin, azithromycin, clarythromycin

Lumbar puncture caveat

patient who is being treated with antibiotic and has a febrile seizure must have lumbar puncture. partially treated meningitis may look clinically benign.

How to describe and identify rashes

patterns of distribution, including where the rash started. Appearance and texture including color, blanching, feel to touch. What season is it? what rashes are common for that season.

Toddler and preschool social/emotional development

play is the primary way they practice social roles.

Eczema herpeticum

primary HSV infection in atopic dermatitis. Umbilicated vesicles over eczema areas, fever, medical emergency because eczema has stopped the body from being able to fight off the virus.

Procalcitonin (PCT)

promising biomarker for differentiating viral from bacterial infections. SBI more likely than other infections to cause increase in CRP.

12 months development: gross motor, fine motor

pulls to stand, stands alone, cruising, rolls ball pincer grasp, points at objects, stacks 2 blocks, claps

Which febrile infants get CXRs?

pulse oximetry <95%, at least 1 clinical sign of pulmonary disease (RR>50/min, rales, rhonchi, retractions, coryza, wheezing, grunting, stridor, nasal flaring, cough). 33% with at least 1 positive had an abnormal CXR.

Multisystem inflammatory immune complex (MIS-C)

rare, clusters of cases. Its usually >2 years, may develop 2 to 4 weeks after covid associated illness. associated with myocarditis, cardiogenic shock, clinically similar to severe kawasaki disease.

DNA acid testing

rely on polymerase chain reaction (PCR) to amplify pathogen specific DNA

More localized rashes

ringworm, tinea versicolor, pityriasis rosea

Well looking and immunizations complete? risk and treatment

risk of bacteremia <1%. UA and urine culture for girls <2 years who meet criteria and for boys < 6 months. antimicrobials: OM or pneumonia: cover for pneumococcus, non-tapeable H flu and moraxella with amoxicillin, amoxicillin + augmentin, ceftriaxone. URI or no focus: no antibiotics in well child (exceptions for incompletely immunized) Pneumococcemia: promptly reassess, if well and afebrile, should at least treat with 1 dose of ceftriazone.

why treat fever with antibiotics in 3 to 36months?

risk of meningitis or other focal bacterial illness. H flu has a much higher risk of meningitis however incidence h flu bacteremia is very low since vaccines even in unvaccinated children.

Rashes that involve painful mucosa or sore throat

scarlet fever, steven johnson syndrome, toxic epidermal necrolysis, HFMD, herpetic gingivostomatitis/whitlow/ herpes labialis

Transient tachypnea of the newborn:

secondary to incomplete evacuation of fetal lung fluid in full-term infants. Tachypnea is secondary to slow absorption of fetal lung fluid. More common in c-section deliveries. Clinical findings: tachypnea, expiratory grunting, intercostal retractions, no abnormal findings on auscultation. May respond to minimal oxygen supplementation. Diagnostic evaluation: chest Xray (pulmonary vascular markings) → Fluid lines along fissures, flat diaphragms, possible pleural fluid. Management: close observation, monitoring of O2 saturation, supplemental oxygen, mechanical ventilation is rarely required. (usually resolves within 24-48 hours).

Respiratory distress syndrome

secondary to surfactant deficiency→ alveolar atelectasis and decreased lung compliance. (formerly called hyaline membrane disease). Most common pulmonary disease in the newborn. Increased incidence at gestational age <33-34 weeks. 50% of neonatal deaths are attributed to RDS or its complications. Diagnostic evaluation: chest radiograph: fine reticular granularity of parenchyma (air bronchograms), blood gas (showing hypoxemia, hypercarbia, mixed metabolic and respiratory acidosis). Management: supportive care, mechanical ventilation, synthetic corticosteroid prior to premature delivery, exogenous surfactant immediately after delivery. Only prevention would be elimination of premature births

RSV: what is it? who is at risk? transmission? incubation? seasonal outbreaks when?

single-stranded RNA virus (types A and B). the leading cause of hospitalizations in children <1 year old and the cause of the majority of bronchiolitis cases. nearly all children are infected at least once by 2 years of age. peak incidence is 2-3 months. increased morbidity and mortality in premature infants and infants with chronic lung disease. seasonal outbreaks: onset november, peak jan-feb, end may. transmission through inoculation of NP or conjunctival mucosa with resp secretions. hand hygiene and contact precautions are essential to reduce spread. incubation: 2-8 days. active shedding: 8 days Risk factors: •bronchopulmonary dysplasia (limited pulmonary functional reserve, distorted airway architecture, and proinflammatorymilieu •Age < 12 weeks •Lack of breastfeeding •Male sex •Congenital heart disease •Immunodeficiency •Community exposures

Ecthyma

skin infection with indurate ulcers. contagious. treat with penicillin.

Spectrum of tinea capitis

slight scale suggesting "Dry scalp", mild eczema, mild seborrhea. papule at base of hair follicle, becomes pustular. Alopecia either patchy or in circles with broken off hairs "salt and pepper". Occipital adenopathy, large fungating tender mass with purulent drainage (Kerion). ID reaction (papular immune reaction), secondary bacterial infection.

rashes with pain

staph scalded skin, kerion (a tinea capitus immune reaction), herpes zoster (shingles)

treatment for intermittent asthma

step 1: SABA prn

Enterovirus (Coxsackie) mangement

supportive care, encourage hand washing, place in precautions

Step 1: intermittent asthma symptoms and lung function

symptoms < 2xs per week, asymptomatic with normal PEF between exacerbations. uses SABA 2xs/week. brief exacerbations of varying intensity, no interference with normal activity. nighttime wakenings/symptoms < or = 2x per month lung function: FEV> 80% predicted, normal FEV between exacerbations

Step 2: mild persistent asthma symptoms and lung function

symptoms > 2 x's per week but less than daily Requires SABA > 2 days/week but not more than once daily Minor affect on daily activity nightime symptoms 3-4xs per month lung function: >80% predicted

viral exanthem

symptoms: fever, nasal congestion, and diarrhea toxic: no Color: red Pruritic: no texture: generalized but patchy coarse papules

herpes zoster (shingles)

symptoms: low grade temp, pain, itching color: erythematous Distribution: dermatome Appearance: grouped vesicles and bullae Pruritic: mild

The best way to identify bacteremia/sepsis in febrile infants?

take a good history, document rectal temp, antipyretics, associated symptoms, feeding playfulness, social smile, vaccination, medical history (are they immunocompromised), duration of fever is a poor predictor. the PE: well vs. ill, change in VS, oral lesions, TMs, lymph, rash, respiratory symptoms, urinary symptoms, musculoskeletal systems, meningeal signs.

Colostrum

the first milk the woman's body produces. Colostrum is higher in protein, minerals, salt, vitamin A, nitrogen, white blood cells, and certain antibodies, and it has less fat and sugar than mature milk

Infants 1 to 3 months: who are toxic and febrile. what do they need?

they have a much higher risk of SBI. Full sepsis workup (LP, CBC, pan culture). admit for IV ampicillin and cefotaxime or cefuroxime, consider Vancomycin if soft tissue infection.

24 months gross and fine motor

throws overhand, runs, climbs, kicks ball, walk up/down stairs, able to pull pants down with assist Builds 7 cube tower, circular scribbling, turns doorknob, turns pages, unbutton/unzips clothing, washes hands (with assistance), uses spoon

definitions toddler and preschooler

toddler= 12-24months Preschooler= 2-5 years

Those at increased risk for drowning

toddlers, adolescents, epilepsy, autism, cardiac arrhythmias

lab work up for Hyperbilirubinemia and treatment

total serum bilirubin, direct bilirubin, CBC with differential, reticulocyte count, blood type and direct antibody test (Coombs) Consider type and crossmatch, albumin, newborn screen results, G6PD Treatment: phototherapy The benefits are to prevent irreversible neurologic disease, noninvasive, relatively well tolerated Cons: parents comment not easy to sleep with blue lights, parents don't get as much skin-to-skin direct contact with baby.

Urticaria

toxic: no other symptoms: none, no fever distribution: patchy and evanescent blanching: yes, pruritic: yes texture: raised flat plaques

scarlet fever rash description

toxic: no fever: yes other symptoms: sore throat, strawberry tongue, petechiae on palate, pruritic distribution: generalized, even blanching: yes color: red (flushed) texture: sand paper

Group A beta hemolytic strep

transmission primarily via upper respiratory secretions or skin invasion. pharyngitis is rare in <3years. peak incidence 3-15 years. resp tract: Tonsillopharyngitis/ pneumonia, Peritonsillar abscess, Cervical lymphadenitis, Retropharyngeal abscess, Otitis media, Mastoiditis, Sinusitis Manifestations: SSTI, impetigo, bacteremia, vaginitis, perianal streptococcal cellulitis, necrotizing fasciitis, streptococcus toxic shock syndrome, post strep glomerulonephritis, rheumatic heart disease.

Pertussis: clinical manifestations/stages and what happens at each.

uncontrollable, violent coughing which often makes it hard to breathe. After coughing fit, loud inspiration. Can last for up to 10+ weeks. Stage 1: catarrhal stage. viral URI with mild cough and coryza (cold like symptoms, clear rhinorrhea). low grade fever, cough gradually increases. lasts 1 to 2 weeks. Stage 2: paroxysmal stage. coughing spells increase in severity, cough is distinctive (long series of coughs between which there is little or no inspiratory effort). The child may gag, develop cyanosis, and appear to be struggling for breath. Posttusive emesis. this phase lasts up to 2 weeks. Stage 3: convalescent stage. cough gradually decreases in severity over several weeks to months.

Whats the leading cause of death in kids 1-5 years old?

unintentional injury (drowning)

Storage of leftover milk from a feeding

use within 2 hours after the baby is done feeding

Cultures, stains, and susceptibilities

usefulness dependent on quality of sample, be aware of community trends and resistance

Pyogenic granuloma: cause, what is it?, treatment

usually caused by low-grade localized infection, trauma, or hormonal factors. vascular lesion that occurs in both mucosa and skin, an overgrowth of tissue due to irritation, physical trauma or hormonal factors. involves gums, skin, nasal septum. Tx: improved oral hygiene, 0.12% cholorhexidine gluconate rinses, surgical excision, cryosurgery, or intralesional injections of corticosteroids. (generally resolves spontaneously, but refer if not resolving to have it removed.)

Intussusception findings and treatment

usually presents during early infancy, often preceded by a URI. Hallmark clinical finding: a sausage shaped mass in the RUQ with emptiness in the LRQ. The mass is hard to detect and is best palpated between spasm of colic, when the infant is quiet. Clinical findings: Vomiting: initially non-bilious proceeds to bilious Pain: colicky, severe, intermittent Stool: blood and mucous→ currant jelly stools Lethargy alternating with crying Abdominal distention frequently is found if obstruction is complete Treatment: gold standard is contrast enema (air or barium). May require surgical correction. Early diagnosis with fluid resuscitation and effective treatment is associated with mortality rate <1%. If left untreated, uniformly fatal in 2-5 days.

Henoch-Schonlein Purpura: etiology, characteristics, diagnostic, treatment

vasculitis or immune reaction of small blood vessels. unknown etiology, may follow strep infections The rash is characteristically below the waist with palpable purpura, bruising and petechiae. accompanying symptoms: abdominal pain, arthralgia and arthritis especially knees and ankles. rectal bleeding and scrotal swelling. Diagnostic: blood and protein in the urine, elevated BP. Normal PLT count and coagulation studies. Serious rare complications: chronic rental disease, intussusception, and brain involvement. Treatment: hospitalization for severe. analgesia and limitation of activity, steroids for severe abdominal symptoms. the usual course is several weeks to months of recurring crops of rash with complete resolution.

Mumps: epidemiology, manifestations, complications

viral illness caused by paramyxovirus. 99% decreased in incidence with vaccination. outbreaks do continue. manifestations: pain, tenderness, and swelling in one or both parotid salivary glands. complications: orchitis, permanent deafness, and rarely encephalitis.

18 months gross and fine motor

walks independently, pushes/pulls toys, throws while aiming. stacks 4-6 blocks, drinks from cup (little spilling), scribbles, remove socks, put blocks in large holes

red flags for Hyperbilirubinemia

yellow coloring extends distally to the knee or lower, yellow color is more intense, lemon yellow to orange yellow. Difficulty in feeding (impaired ability to suck or feed), vomiting. Lethargy (hard to wake up infant for feeding), irritability/ difficult to console, arching of neck or body. Fever, high-pitched cry.

Weaning baby from breastmilk

•Breastfeeding is encouraged for 1 yr, but circumstances may dictate a different choice for families. Natural weaning occurs once other foods are introduced, and the infant begins to self-feed Weaning should be a gradual process. Only pump when uncomfortable, not to empty. This gradually decreases milk supply. Some mothers grieve the loss of breastfeeding experience whether it's planned or unplanned. To prevent premature weaning, providers should look for factors associated, such as post-partum depression, perception of insufficient breastmilk, and maternal dissatisfaction

important assessment findings for peds asthma

•Wheezing (heretophonous?) •Coughing (continuous, persistent?) •Auscultation: Prolonged expiratory phase, Rhonchi at bases, Diminished breath sounds •Evidence of respiratory distress •Tachycardia, hypertension, hypotension, pulsus paradoxus •Cyanosis (circumoral, nail beds) is late and very concerning •Associated findings (rhinitis, dermatitis, sinusitis)

pertussis complications

◦1% mortality in infants <6 months of age ◦Apnea ◦Bacterial pneumonia ◦Seizures ◦Encephalopathy ◦Epistaxis ◦Subconjunctival hemorrhage ◦Syncope ◦Rib fractures ◦Incontinence ◦Death

Differential diagnoses for fever, tachypnea, fine crackles/dim breath sounds, mild subcostal retractions, nasal flaring, cough, fever?

◦Pneumonia: Viral, bacterial, parasitic ◦Foreign body ◦Bronchiolitis ◦Congestive heart failure ◦Pulmonary abscess ◦Pulmonary effusion ◦Acute bronchiectasis


Conjuntos de estudio relacionados

Nightingale Anatomy Final Chapter 13 & 14

View Set

Strategic Management and Strategic Competitiveness

View Set

Principles of Macroeconomics (Ch. 12, 13, 14, 15)

View Set

Chapter 7 Segmentation, Target Marketing, and Positioning

View Set

Chapter 18 - Vital Signs and Measurement

View Set